Формула угловой коэффициент касательной к графику функции: Урок 14. геометрический смысл производной — Алгебра и начала математического анализа — 11 класс

Содержание

Урок 14. геометрический смысл производной — Алгебра и начала математического анализа — 11 класс

Алгебра и начала анализа, 11 класса.

Урок №14. Геометрический смысл производной.

Перечень вопросов, рассматриваемых в теме

1) Геометрический смысл производной;

2) Алгоритм нахождения касательной к графику функции в точке;

3) Сравнение производных заданной функции по ее графику в различных точках.

Глоссарий по теме

Число k= tgα называется угловым коэффициентом прямой, а угол α – углом между этой прямой и осью Ох.

Геометрический смысл производной. Производная в точке x0 равна угловому коэффициенту касательной к графику функции y = f(x) в этой точке.

Основная литература:

Колягин Ю.М., Ткачева М.В, Федорова Н.Е. и др., под ред. Жижченко А.Б. Алгебра и начала математического анализа (базовый и профильный уровни) 10 кл.– М.: Просвещение, 2014.

Дополнительная литература:

Шабунин М.И., Ткачева М.В., Федорова Н.Е. Дидактические материалы Алгебра и начала математического анализа (базовый и профильный уровни) 10 кл. – М.: Просвещение, 2017.

Теоретический материал для самостоятельного изучения

Напомним, что графиком линейной функции у=кх + b является прямая.

Число k= tgα называется угловым коэффициентом прямой, а угол α – углом между этой прямой и осью Ох.

Если k>0, то 0<α< π/2, в этом случае функция возрастает

Если k<0, то — π/2<α<0, в этом случае функция убывает

Геометрический смысл производной. Производная в точке x0 равна угловому коэффициенту касательной к графику функции y = f(x) в этой точке.

Рассмотрим график функции f ( ):

Из рисунка видно, что для любых двух точек A и B графика функции: f(x0+Δx)/f(x0)Δx=tgα, где  — угол наклона секущей 

AB.  

Таким образом, разностное отношение равно угловому коэффициенту секущей. 

Если зафиксировать точку A и двигать по направлению к ней точку B, то Δx неограниченно уменьшается и приближается к 0, а секущая АВ приближается к касательной АС

Следовательно, предел разностного отношения равен угловому коэффициенту касательной в точке A.

Отсюда следует:

производная функции в точке есть угловой коэффициент касательной к графику этой функции в этой точке.

В этом и состоит геометрический смысл производной.

Уравнение касательной к графику функции y=f(x) в точке x0:

Примеры и разбор решения заданий тренировочного модуля

№1. Составить уравнение касательной к графику функции y=x+e-2x, параллельной прямой y=-x

Решение:

Угловой коэффициент касательной равен значению производной в точке касания x0. Т.к. касательная параллельна прямой y=-x, значит ее угловой коэффициент равен –1. Таким образом, f'(x0) = -1.

Уравнение касательной:

Уравнение касательной: y=1-1(x-0) = 1-x

Ответ: y=1-x.

№2. На параболе у=х2-2х-8 найти точку М, в которой касательная к ней параллельна прямой 4х+у+4=0.

Решение:

Определим угловой коэффициент касательной к параболе у=х2-2х-8:

k =у’=(х2-2х-8)’=2х-2.

Найдем угловой коэффициент прямой 4х+у+4=0:

у=-4х-4, k =-4.

Касательная к параболе и данная прямая по условию параллельны. Следовательно, их угловые коэффициенты равны, т.е.

2х-2=-4;

х=-1 – абсцисса точки касания.

Ординату точки касания М вычислим из уравнения данной параболы у=х2-2х-8, т.е.

у(-1)=(-1)2-2(-1)-8=-5, М(-1;-5).

Ответ: М(-1;-5).

Формула касательной к графику через производную. Как найти угловой коэффициент

Касательная – это прямая, проходящая через точку кривой и совпадающая с ней в этой точке с точностью до первого порядка (рис. 1).

Другое определение : это предельное положение секущей при Δx →0.

Пояснение : Возьмем прямую, пересекающую кривую в двух точках: А и b (см.рисунок). Это секущая. Будем поворачивать ее по часовой стрелке до тех пор, пока она не обретет только одну общую точку с кривой. Так мы получим касательную.

Строгое определение касательной:

Касательная к графику функции f , дифференцируемой в точке x о , — это прямая, проходящая через точку (x о ; f (x о )) и имеющая угловой коэффициент f ′(x о ).

Угловой коэффициент имеет прямая вида

y = kx + b . Коэффициент k и является угловым коэффициентом этой прямой.

Угловой коэффициент равен тангенсу острого угла, образуемого этой прямой с осью абсцисс:

Здесь угол α – это угол между прямой y = kx + b и положительным (то есть против часовой стрелки) направлением оси абсцисс. Он называется углом наклона прямой (рис.1 и 2).

Если угол наклона прямой y = kx + b острый, то угловой коэффициент является положительным числом. График возрастает (рис.1).

Если угол наклона прямой y = kx + b тупой, то угловой коэффициент является отрицательным числом. График убывает (рис.2).

Если прямая параллельна оси абсцисс, то угол наклона прямой равен нулю. В этом случае угловой коэффициент прямой тоже равен нулю (так как тангенс нуля есть ноль). Уравнение прямой будет иметь вид y = b (рис.3).

Если угол наклона прямой равен 90º (π/2), то есть она перпендикулярна оси абсцисс, то прямая задается равенством x = c , где c – некоторое действительное число (рис.4).

Уравнение касательной к графику функции y = f (x ) в точке x о :


Пример : Найдем уравнение касательной к графику функции f (x ) = x 3 – 2x 2 + 1 в точке с абсциссой 2.

Решение .

Следуем алгоритму.

1) Точка касания x о равна 2. Вычислим f (x о ):

f (x о ) = f (2) = 2 3 – 2 ∙ 2 2 + 1 = 8 – 8 + 1 = 1

2) Находим f ′(x ). Для этого применяем формулы дифференцирования, изложенные в предыдущем разделе. Согласно этим формулам,

х 2 = 2х , а х 3 = 3х 2 . Значит:

f ′(x ) = 3х 2 – 2 ∙ 2х = 3х 2 – 4х .

Теперь, используя полученное значение f ′(x ), вычислим f ′(x о ):

f ′(x о ) = f ′(2) = 3 ∙ 2 2 – 4 ∙ 2 = 12 – 8 = 4.

3) Итак, у нас есть все необходимые данные: x о = 2, f (x о ) = 1, f ′(x о ) = 4. Подставляем эти числа в уравнение касательной и находим окончательное решение:

у = f (x о ) + f ′(x о ) (x – x о ) = 1 + 4 ∙ (х – 2) = 1 + 4х – 8 = –7 + 4х = 4х – 7.

Ответ : у = 4х – 7.

В этой статье мы разберем все типы задач на нахождение

Вспомним геометрический смысл производной : если к графику функции в точке проведена касательная, то коэффициент наклона касательной (равный тангенсу угла между касательной и положительным направлением оси ) равен производной функции в точке .


Возьмем на касательной произвольную точку с координатами :


И рассмотрим прямоугольный треугольник :


В этом треугольнике

Отсюда

Это и есть уравнение касательной, проведенной к графику функции в точке .

Чтобы написать уравнение касательной, нам достаточно знать уравнение функции и точку, в которой проведена касательная. Тогда мы сможем найти и .

Есть три основных типа задач на составление уравнения касательной.

1. Дана точка касания

2. Дан коэффициент наклона касательной, то есть значение производной функции в точке .

3. Даны координаты точки, через которую проведена касательная, но которая не является точкой касания.

Рассмотрим каждый тип задач.

1 . Написать уравнение касательной к графику функции в точке .

.

б) Найдем значение производной в точке . Сначала найдем производную функции

Подставим найденные значения в уравнение касательной:

Раскроем скобки в правой части уравнения. Получим:

Ответ: .

2 . Найти абсциссы точек, в которых касательные к графику функции параллельны оси абсцисс.

Если касательная параллельна оси абсцисс, следовательно угол между касательной и положительным направлением оси равен нулю, следовательно тангенс угла наклона касательной равен нулю. Значит, значение производной функции в точках касания равно нулю.

а) Найдем производную функции .

б) Приравняем производную к нулю и найдем значения , в которых касательная параллельна оси :

Приравняем каждый множитель к нулю, получим:

Ответ: 0;3;5

3 . Написать уравнения касательных к графику функции , параллельных прямой .

Касательная параллельна прямой . Коэффициент наклона этой прямой равен -1. Так как касательная параллельна этой прямой, следовательно, коэффициент наклона касательной тоже равен -1. То есть мы знаем коэффициент наклона касательной , а, тем самым, значение производной в точке касания .

Это второй тип задач на нахождение уравнения касательной.

Итак, у нас дана функция и значение производной в точке касания.

а) Найдем точки, в которых производная функции равна -1.

Сначала найдем уравнение производной.

Приравняем производную к числу -1.

Найдем значение функции в точке .

(по условию)

.

б) Найдем уравнение касательной к графику функции в точке .

Найдем значение функции в точке .

(по условию).

Подставим эти значения в уравнение касательной:

.

Ответ:

4 . 2}»>. Мы получили под корнем отрицательное число, равенство не верно, и точка не принадлежит графику функции и не является точкой касания.

Это последний тип задач на нахождение уравнения касательной. Первым делом нам нужно найти абсциссу точки касания .

Найдем значение .

Пусть — точка касания. Точка принадлежит касательной к графику функции . Если мы подставим координаты этой точки в уравнение касательной, то получим верное равенство:

.

Значение функции в точке равно .

Найдем значение производной функции в точке .

Сначала найдем производную функции . Это .

Производная в точке равна .

Подставим выражения для и в уравнение касательной. Получим уравнение относительно :

Решим это уравнение.

Сократим числитель и знаменатель дроби на 2:

Приведем правую часть уравнения к общему знаменателю. Получим:

Упростим числитель дроби и умножим обе части на — это выражение строго больше нуля. 2} {8-3x_0>=0} }}{ }»>

Решим первое уравнение.

Решим квадратное уравнение, получим

Второй корень не удовлетворяет условию title=»8-3x_0>=0″>, следовательно, у нас только одна точка касания и её абсцисса равна .

Напишем уравнение касательной к кривой в точке . Для этого подставим значение в уравнение — мы его уже записывали.

Ответ:
.

Вам понадобится

  • — математический справочник;
  • — тетрадь;
  • — простой карандаш;
  • — ручка;
  • — транспортир;
  • — циркуль.

Инструкция

Примите к сведению, что график дифференцируемой функции f(x) в точке х0 не имеет различий с отрезком касательной. Поэтому он достаточно близким к отрезку l, к проходящему через точки (х0; f(х0)) и (х0+Δx; f(x0 + Δx)). Чтобы задать прямую, проходящую через точку А с коэффициентами (х0; f(х0)), укажите ее угловой коэффициент. При этом он равен Δy/Δx секущей касательной (Δх→0) , а также стремится к числу f‘(x0).

Если значений f‘(x0) не существует, то, касательной нет, или же она проходит вертикально. Исходя из этого, производной функции в точке х0 объясняется существованием невертикальной касательной, которая соприкасается с графиком функции в точке (х0, f(х0)). В данном случае угловой коэффициент касательной равняется f»(х0). Становится понятен геометрический производной, то есть углового коэффициента касательной.

То есть для того чтобы найти угловой коэффициент касательной, нужно найти значение производной функции в точке касания. Пример: найти угловой коэффициент касательной к функции у = х³ в точке с абсциссой Х0 = 1. Решение: Найдите производную данной функции у΄(х) = 3х²; найдите значение производной в точке Х0 = 1. у΄(1) = 3 × 1² = 3. Угловой коэффициент касательной в точке Х0 = 3.

Начертите на рисунке дополнительные касательные таким образом, чтобы они соприкасались с графиком функции в точках: x1, х2 и х3. Отметьте углы, которые образуются данными касательными с осью абсцисс (угол отсчитывается в положительном направлении — от оси до касательной прямой). Например, угол α1 будет острым, же (α2) – тупой, ну а третий (α3) будет равняться нулю, так как проведенная касательная прямая является параллельной оси ОХ. В этом случае тангенс тупого угла есть отрицательное значение, а тангенс острого угла – положительное, при tg0 и результат равен нулю.

Касательной к заданной окружности называется прямая линяя, которая имеет только одну общую точку с этой окружностью. Касательная к окружности всегда перпендикулярна его радиусу, проведённому к точке касания. Если две касательные проведены из одной точки, не принадлежащей окружности, то расстояния от этой точки до точек касания всегда будет одинаковым. Касательные к окружностям строятся разными способами, в зависимости от их расположения относительно друг друга.

Инструкция

Построение касательной к одной окружности.
1. Строится окружность радиуса R и берётся A, которую будет проходить касательная.
2. Строится окружность с центром в середине отрезка OA и радиусам равным этого отрезка.
3. Пересечения двух точками касания касательных проведённых через точку A к заданной окружности.

Внешняя касательная к двум окружностям .

2. Проводится окружность радиусом R – r с центром в точке O.
3. К полученной окружности проводится касательная из O1, точка касания обозначена M.
4. Радиус R проходящий через точку M на точку T – точку касания окружности.
5. Через центр O1 малой окружности проводится радиус r параллельно R большой окружности. Радиус r указывает на точку T1 – точку касания малой окружности.
окружностям .

Внутренняя касательная к двум окружностям .
1. Строятся две окружности радиусом R и r.
2. Проводится окружность радиусом R + r с центром в точке O.
3. К полученной окружности проводится касательная из точки O1, точка касания обозначена буквой M.
4. Луч OM пересекает первую окружность в точке T – в точке касания большой окружности.
5. Через центр O1 малой окружности проводится радиус r параллельно лучу OM. Радиус r указывает на точку T1 – точку касания малой окружности.
6. Прямая TT1 – касательная к заданным окружностям .

Источники:

  • внутренняя касательная

Угловой шкаф – идеальный вариант для пустующих углов в квартире. Кроме того, конфигурация угловых шкаф ов придает интерьеру классическую атмосферу. В качестве отделки угловых шкаф ов может быть использован любой материал, который подходит для этой цели.

Вам понадобится

Инструкция

Вырежьте из фанеры или ДВП шаблон шириной 125 мм, длиной 1065 мм. Кромки необходимо запилить под углом 45 градусов. По готовому шаблону определите размеры боковых стенок, а так же место, где будет расположен шкаф .

Крышку соедините с боковыми стенками и треугольными полками. Крепление крышки должно происходить к верхним кромкам боковых стенок при помощи шурупов. Для прочности конструкции дополнительно используют клей. Полки прикрепите к планкам.

Наклоните пильный диск под углом 45 градусов и скосите по направляющей планке переднюю кромку боковых стенок. Неподвижные полки прикрепите к планкам МДФ. Соедините боковые стенки при помощи шурупов. Следите за тем, чтобы не было щелей.

В стене сделайте отметки, между которыми поставьте каркас углового шкаф а. С помощью шурупов прикрепите шкаф к стене. Длина дюбеля должна быть 75 мм.

Из цельной плиты МДФ выпилите лицевую рамку. С помощью дисковой пилы вырежьте в ней проемы, используя линейку. Допилите углы.

Найдите значение абсциссы точки касания, которую обозначаются буквой «а». Если она совпадает с заданной точкой касательной, то «а» будет ее х-координате. Определите значение функции f(a), подставив в уравнение функции величину абсциссы.

Определите первую производную уравнения функции f’(x) и подставьте в него значение точки «а».

Возьмите общее уравнение касательной, которое определяется как y = f(a) = f (a)(x – a), и подставьте в него найденные значения a, f(a), f «(a). В результате будет найдено решение графика и касательной.

Решите задачу иным способом, если заданная точка касательной не совпала с точкой касания. В этом случае необходимо в уравнение касательной вместо цифр подставить «а». После этого вместо букв «х» и «у» подставьте значение координат заданной точки. Решите получившееся уравнение, в котором «а» является неизвестной. Поставьте полученное значение в уравнение касательной.

Составьте уравнение касательной с буквой «а», если в условии задачи задано уравнение функции и уравнение параллельной линии относительно искомой касательной. После этого необходимо производную функции , чтобы координату у точки «а». Подставьте соответствующее значение в уравнение касательной и решите функцию.

При составлении уравнения касательной к графику функции используется понятие «абсцисса точки касания». Данная величина может задаваться изначально в условиях задачи или же ее необходимо определять самостоятельно.

Инструкция

Начертите на листе в клеточку оси координат х и у. Изучите заданное уравнение для графика функции. Если оно является , то достаточно два значения для параметра у при любых х, после чего построить найденные точки на оси координат и соединить их линией. Если же график нелинейный, то составьте таблицу зависимости у от х и подберите как минимум пять точек для построения графика.

Определите значение абсциссы точки касания для случая, когда заданная точка касательной не совпадает с графиком функции. Задаем третий параметр буквой «а».

Запишите уравнение функции f(a). Для этого в исходное уравнение вместо х подставьте а. Найдите производную функции f(x) и f(a). Подставьте необходимые данные в общее уравнение касательной, которое имеет вид: y = f(a) + f «(a)(x – a). В результате получить уравнение, которое из трех неизвестных параметров.

Подставьте в него вместо х и у координаты заданной точки, через которую проходит касательная. После этого найдите решение полученного уравнения для всех а. Если оно является квадратным, то будет два значения абсциссы точки касания. Это , что касательная проходит два раза возле графика функции.

Нарисуйте график заданной функции и , которые заданы по условию задачи. В этом случае необходимо также задать неизвестный параметр а и подставить его в уравнение f(a). Приравняйте производную f(a) к производной уравнения параллельной прямой. Данное выходит из условия параллельности двух . Найдите корни полученного уравнения, которые будут являться абсциссами точки касания.

Прямая y=f(x) будет касательной к изображенному на рисунке графику в точке х0 в том случае, если она проходит через точку с координатами (х0; f(x0)) и обладает угловым коэффициентом f»(x0). Найти такой коэффициент, зная особенности касательной, несложно.

Вам понадобится

  • — математический справочник;
  • — простой карандаш;
  • — тетрадь;
  • — транспортир;
  • — циркуль;
  • — ручка.

Инструкция

Если значения f‘(x0) не существует, то либо касательной нет, либо она проходит вертикально. Ввиду этого, наличие производной функции в точке х0 обусловлено существованием невертикальной касательной, соприкасающейся с графиком функции в точке (х0, f(х0)). В этом случае угловой коэффициент касательной равен будет f»(х0). Таким образом, становится ясен геометрический смысл производной – расчет углового коэффициента касательной.

Определите общую . Подобного рода сведения можно получить, обратившись к данным переписи населения. Для определения общих коэффициентов рождаемости, смертности, брачности и разводимости вам понадобится найти произведение общей населения и расчетного периода. Получившееся число запишите в знаменатель.

Поставьте на числителя показатель, соответствующий искомому относительному. Например, если перед вами стоит определить общий коэффициент рождаемости, то на месте числителя должно находиться число, отражающее общее количество рожденных за интересующий вас период. Если вашей целью является уровня смертности или брачности, то на место числителя поставьте число умерших в расчетный период или число вступивших в брак, соответственно.

Умножьте получившееся число на 1000. Это и будет искомый вами общий коэффициент. Если же перед вами стоит задача найти общий коэффициент прироста, то вычтите из коэффициента рождаемости коэффициент смертности.

Видео по теме

Источники:

  • Общие коэффициенты естественного движения населения

Главным показателем эффективности экстракции является коэффициент распределения . Он считается по формуле: Со/Св, где Со – концентрация извлекаемого вещества в органическом растворителе (экстракторе), а Св – концентрация этого же вещества в воде, после наступления равновесия. Как можно опытным путем найти коэффициент распределения?

Касательная — это прямая , которая касается графика функции в одной точке и все точки которой находятся на наименьшем расстоянии от графика функции. Поэтому касательная проходит касательно графика функции под определённым углом и не могут проходить через точку касания несколько касательных под разными углами. Уравнения касательной и уравнения нормали к графику функции составляются с помощью производной.

Уравнение касательной выводится из уравнения прямой .

Выведем уравнение касательной, а затем — уравнение нормали к графику функции.

y = kx + b .

В нём k — угловой коэффициент.

Отсюда получаем следующую запись:

y y 0 = k (x x 0 ) .

Значение производной f «(x 0 ) функции y = f (x ) в точке x 0 равно угловому коэффициенту k = tgφ касательной к графику функции, проведённой через точку M 0 (x 0 , y 0 ) , где y 0 = f (x 0 ) . В этом состоит геометрический смысл производной .

Таким образом, можем заменить k на f «(x 0 ) и получить следующее уравнение касательной к графику функции :

y y 0 = f «(x 0 )(x x 0 ) .

В задачах на составление уравнения касательной к графику функции (а мы уже скоро к ним перейдём) требуется привести получившееся по вышеприведённой формуле уравнение к уравнению прямой в общем виде . Для этого нужно все буквы и числа перенести в левую часть уравнения, а в правой части оставить ноль.

Теперь об уравнении нормали. Нормаль — это прямая, проходящая через точку касания к графику функции перпендикулярно касательной. Уравнение нормали :

(x x 0 ) + f «(x 0 )(y y 0 ) = 0

Для разминки первый же пример прелагается решить самостоятельно, а затем посмотреть решение. Есть все основания надеяться, что для наших читателей эта задача не будет «холодным душем».

Пример 0. Составить уравнение касательной и уравнение нормали к графику функции в точке M (1, 1) .

Пример 1. Составить уравнение касательной и уравнение нормали к графику функции , если абсцисса точки касания .

Найдём производную функции:

Теперь у нас есть всё, что требуется подставить в приведённую в теоретической справке запись, чтобы получить уравнение касательной. Получаем

В этом примере нам повезло: угловой коэффициент оказался равным нулю, поэтому отдельно приводить уравнение к общему виду не понадобилось. Теперь можем составить и уравнение нормали:

На рисунке ниже: график функции бордового цвета, касательная зелёного цвета, нормаль оранжевого цвета.

Следующий пример — тоже не сложный: функция, как и в предыдущем, также представляет собой многочлен, но угловой коэффициен не будет равен нулю, поэтому добавится ещё один шаг — приведение уравнения к общему виду.

Пример 2.

Решение. Найдём ординату точки касания:

Найдём производную функции:

.

Найдём значение производной в точке касания, то есть угловой коэффициент касательной:

Подставляем все полученные данные в «формулу-болванку» и получаем уравнение касательной:

Приводим уравнение к общему виду (все буквы и числа, отличные от нуля, собираем в левой части, а в правой оставляем ноль):

Составляем уравнение нормали:

Пример 3. Составить уравнение касательной и уравнение нормали к графику функции , если абсцисса точки касания .

Решение. Найдём ординату точки касания:

Найдём производную функции:

.

Найдём значение производной в точке касания, то есть угловой коэффициент касательной:

.

Находим уравнение касательной:

Перед тем, как привести уравнение к общему виду, нужно его немного «причесать»: умножить почленно на 4. Делаем это и приводим уравнение к общему виду:

Составляем уравнение нормали:

Пример 4. Составить уравнение касательной и уравнение нормали к графику функции , если абсцисса точки касания .

Решение. Найдём ординату точки касания:

.

Найдём производную функции:

Найдём значение производной в точке касания, то есть угловой коэффициент касательной:

.

Получаем уравнение касательной:

Приводим уравнение к общему виду:

Составляем уравнение нормали:

Распространённая ошибка при составлении уравнений касательной и нормали — не заметить, что функция, данная в примере, — сложная и вычислять её производную как производную простой функции. Следующие примеры — уже со сложными функциями (соответствующий урок откроется в новом окне).

Пример 5. Составить уравнение касательной и уравнение нормали к графику функции , если абсцисса точки касания .

Решение. Найдём ординату точки касания:

Внимание! Данная функция — сложная, так как аргумент тангенса (2x ) сам является функцией. Поэтому найдём производную функции как производную сложной функции.

С понятием касательной к графику функции вы уже знакомы. График дифференцируемой в точке х 0 функции f вблизи х 0 практически не отличается от отрезка касательной, а значит, он близок к отрезку секущей l, проходящей через точки (х 0 ; f (х 0)) и (х 0 +Δx; f (x 0 + Δx)). Любая из таких секущих проходит через точку А (х 0 ; f (х 0)) графика (рис. 1). Для того чтобы однозначно задать прямую, проходящую через данную точку A, достаточно указать ее угловой коэффициент. Угловой коэффициент Δy/Δx секущей при Δх→0 стремится к числу f ‘(x 0) (его мы примем за угловой коэффициент касательной) Говорят, что касательная есть предельное положение секущей при Δх→0 .

Если же f’(х 0) не существует, то касательная либо не существует (как у функции у = |x| в точке (0; 0), см. рис.), либо вертикальна (как у графика функции в точке (0; 0), рис.2).

Итак, существование производной функции f в точке хо эквивалентно существованию (невертикальной) касательной в точке (х 0 , f (х 0)) графика, при этом угловой коэффициент касательной равен f» (х 0). В этом состоитгеометрический смысл производной

Касательная к графику дифференцируемой в точке xо функции f — это прямая, проходящая через точку (x 0 ; f (x 0)) и имеющая угловой коэффициент f ‘(х 0).

Проведем касательные к графику функции f в точках x 1 , х 2 , х 3 (рис. 3) и отметим углы, которые они образуют с осью абсцисс. (Это угол, отсчитываемый в положительном направлении от положительного направления оси до прямой.) Мы видим, что угол α 1 острый, угол α 3 тупой, а угол α 2 равен нулю, так как прямая l параллельна оси Ох. Тангенс острого угла положителен, тупого — отрицателен, tg 0 = 0. Поэтому

F»(x 1)>0, f’(x 2)=0, f’(x 3)
Построение касательных в отдельных точках позволяет более точно строить эскизы графиков. Так, например, для построения эскиза графика функции синус предварительно находим, что в точках 0; π/2 и π производная синуса равна 1; 0 и -1 соответственно. Построим прямые, проходящие через точки (0; 0), (π/2,1) и (π, 0) с угловыми коэффициентами 1, 0 и -1 соответственно (рис. 4) Остается вписать в полученную трапецию, образованную этими прямыми и прямой Ох, график синуса так, чтобы при х, равном 0, π/2 и π, он касался соответствующих прямых.

Отметим, что график синуса в окрестности нуля практически не отличим от прямой у = х. Пусть, например, масштабы по осям выбраны так, что единице соответствует отрезок в 1см. Имеем sin 0,5 ≈ 0,479425, т. е. |sin 0,5 — 0,5| ≈ 0,02, и в выбранном масштабе это соответствует отрезку длиной 0,2 мм. Поэтому график функции y = sin x в интервале (-0,5; 0,5) будет отклоняться (в вертикальном направлении) от прямой у = х не более чем на 0,2 мм, что примерно соответствует толщине проводимой линии.

Угловой коэффициент касательной

 

В школьной геометрии касательная к окружности определяется как прямая, которая имеет с окружностью только одну общую точку — точку касания.

 

В высшей математике касательная, проведённая в точке M — это предельное положение секущей MN.

 

 

 

 

 

Пусть дан график функции y=f(x).

Отменим на нём точку M, в которой существует касательная к графику, не параллельная оси абсцисс.

M(xо;f(xо)).

Зададим значению аргумента приращение Δx.

Значению аргумента xо+Δx на графике функции y=f(x) соответствует точка N(xо+Δx;f(xо+Δx)).

Угловой коэффициент секущей MN

равен

   

Если приращение стремится к нулю (Δx→0), то секущая MN, поворачиваясь вокруг точки M, стремится к касательной, проведённой в точке M.

Если k — угловой коэффициент этой касательной, то

   

то есть

   

Если касательная к графику функции в некоторой точке параллельна оси абсцисс, то угловой коэффициент такой касательной равен нулю.

 

Геометрический смысл производной:

Угловой коэффициент касательной к графику функции y=f(x), проведённой в точке с абсциссой xо, равен значению производной в точке касания:

k=f'(xо)

 

Так как угловой коэффициент прямой равен тангенсу угла α между прямой и положительным направлением оси абсцисс, то tgα=f'(xо).

 

Примеры.

№1

Найти угловой коэффициент касательной к графику функции

   

в точке с абсциссой xо=-2.
Решение:

   

   

Ответ: 15.

№2

Найти абсциссу точки графика функции

   

в которой угловой коэффициент касательной к графику равен -9.

Решение:

   

   

По условию, k=-9. Отсюда

   

   

Ответ: 3.

Угловой коэффициент касательной к графику функции положительный.

Угловой коэффициент касательной как тангенс угла наклона

Научитесь брать производные от функций. Производная характеризует скорость изменения функции в определенной точке, лежащей на графике этой функции. В данном случае графиком может быть как прямая, так и кривая линия. То есть производная характеризует скорость изменения функции в конкретный момент времени. Вспомните общие правила, по которым берутся производные, и только потом переходите к следующему шагу.

  • Прочитайте статью .
  • Как брать простейшие производные, например, производную показательного уравнения, описано . Вычисления, представленные в следующих шагах, будут основаны на описанных в ней методах.

Научитесь различать задачи, в которых угловой коэффициент требуется вычислить через производную функции. В задачах не всегда предлагается найти угловой коэффициент или производную функции. Например, вас могут попросить найти скорость изменения функции в точке А(х,у). {2}+6x} в точке А(4,2) равен 22.

  • Если возможно, проверьте полученный ответ на графике. Помните, что угловой коэффициент можно вычислить не в каждой точке. Дифференциальное исчисление рассматривает сложные функции и сложные графики, где угловой коэффициент можно вычислить не в каждой точке, а в некоторых случаях точки вообще не лежат на графиках. Если возможно, используйте графический калькулятор, чтобы проверить правильность вычисления углового коэффициента данной вам функции. В противном случае проведите касательную к графику в данной вам точке и подумайте, соответствует ли найденное вами значение углового коэффициента тому, что вы видите на графике.

    • Касательная будет иметь тот же угловой коэффициент, что и график функции в определенной точке. Для того, чтобы провести касательную в данной точке, двигайтесь вправо/влево по оси Х (в нашем примере на 22 значения вправо), а затем вверх на единицу по оси Y. Отметьте точку, а затем соедините ее с данной вам точкой. В нашем примере соедините точки с координатами (4,2) и (26,3).
  • В математике одним из параметров, описывающих положение прямой на декартовой плоскости координат, является угловой коэффициент этой прямой. Этот параметр характеризует наклон прямой к оси абцисс. Чтобы понять, как найти угловой коэффициент, сначала вспомним общий вид уравнения прямой в системе координат XY.

    В общем виде любую прямую можно представить выражением ax+by=c, где a, b и c — произвольные действительные числа, но обязательно a 2 + b 2 ≠ 0.

    Подобное уравнение с помощью несложных преобразований можно довести до вида y=kx+d, в котором k и d — действительные числа. Число k является угловым коэффициентом, а само уравнение прямой подобного вида называется уравнением с угловым коэффициентом. Получается, что для нахождения углового коэффициента, необходимо просто привести исходное уравнение к указанному выше виду. Для более полного понимания рассмотрим конкретный пример:

    Задача: Найти угловой коэффициент линии, заданной уравнением 36x — 18y = 108

    Решение: Преобразуем исходное уравнение.

    Ответ: Искомый угловой коэффициент данной прямой равен 2.

    В случае, если в ходе преобразований уравнения мы получили выражение типа x = const и не можем в результате представить y в виде функции x, то мы имеем дело с прямой, параллельной оси Х. Угловой коэффициент подобной прямой равен бесконечности.

    Для прямых, которых выражены уравнением типа y = const, угловой коэффициент равняется нулю. Это характерно для прямых, параллельных оси абцисс. Например:

    Задача: Найти угловой коэффициент линии, заданной уравнением 24x + 12y — 4(3y + 7) = 4

    Решение: Приведем исходное уравнение к общему виду

    24x + 12y — 12y + 28 = 4

    Из полученного выражения выразить y невозможно, следовательно угловой коэффициент данной прямой равен бесконечности, а сама прямая будет параллельна оси Y.

    Геометрический смысл

    Для лучшего понимания обратимся к картинке:

    На рисунке мы видим график функции типа y = kx. Для упрощения примем коэффициент с = 0. В треугольнике ОАВ отношение стороны ВА к АО будет равно угловому коэффициенту k. Вместе с тем отношение ВА/АО — это тангенс острого угла α в прямоугольном треугольнике ОАВ. Получается, что угловой коэффициент прямой равняется тангенсу угла, который составляет эта прямая с осью абцисс координатной сетки.

    Решая задачу, как найти угловой коэффициент прямой, мы находим тангенс угла между ней и осью Х сетки координат. Граничные случаи, когда рассматриваемая прямая параллельна осям координат, подтверждают вышенаписанное. Действительно для прямой, описанной уравнением y=const, угол между ней и осью абцисс равен нулю. Тангенс нулевого угла также равен нулю и угловой коэффициент тоже равен нулю.

    Для прямых, перпендикулярных оси абцисс и описываемых уравнением х=const, угол между ними и осью Х равен 90 градусов. Тангенс прямого угла равен бесконечности, так же и угловой коэффициент подобных прямых равен бесконечности, что подтверждает написанное выше.

    Угловой коэффициент касательной

    Распространенной, часто встречающейся на практике, задачей является также нахождение углового коэффициента касательной к графику функции в некоторой точке. Касательная — это прямая, следовательно к ней также применимо понятие углового коэффициента.

    Чтобы разобраться, как найти угловой коэффициент касательной, нам будет необходимо вспомнить понятие производной. Производная от любой функции в некоторой точке — это константа, численно равная тангенсу угла, который образуется между касательной в указанной точке к графику этой функции и осью абцисс. Получается, что для определения углового коэффициента касательной в точке x 0 , нам необходимо рассчитать значение производной исходной функции в этой точке k = f»(x 0). Рассмотрим на примере:

    Задача: Найти угловой коэффициент линии, касательной к функции y = 12x 2 + 2xe x при х = 0,1.

    Решение: Найдем производную от исходной функции в общем виде

    y»(0,1) = 24 . 0,1 + 2 . 0,1 . e 0,1 + 2 . e 0,1

    Ответ: Искомый угловой коэффициент в точке х = 0,1 равен 4,831

    Теме «Угловой коэффициент касательной как тангенс угла наклона» в аттестационном экзамене отводится сразу несколько заданий. В зависимости от их условия, от выпускника может требоваться как полный ответ, так и краткий. При подготовке к сдаче ЕГЭ по математике ученику обязательно стоит повторить задачи, в которых требуется вычислить угловой коэффициент касательной.

    Сделать это вам поможет образовательный портал «Школково». Наши специалисты подготовили и представили теоретический и практический материал максимально доступно. Ознакомившись с ним, выпускники с любым уровнем подготовки смогут успешно решать задачи, связанные с производными, в которых требуется найти тангенс угла наклона касательной.

    Основные моменты

    Для нахождения правильного и рационального решения подобных заданий в ЕГЭ необходимо вспомнить базовое определение: производная представляет собой скорость изменения функции; она равна тангенсу угла наклона касательной, проведенной к графику функции в определенной точке. Не менее важно выполнить чертеж. Он позволит найти правильное решение задач ЕГЭ на производную, в которых требуется вычислить тангенс угла наклона касательной. Для наглядности лучше всего выполнить построение графика на плоскости ОХY.

    Если вы уже ознакомились с базовым материалом на тему производной и готовы приступить к решению задач на вычисление тангенса угла наклона касательной, подобных заданиям ЕГЭ, сделать это можно в режиме онлайн. Для каждого задания, например, задач на тему «Связь производной со скоростью и ускорением тела» , мы прописали правильный ответ и алгоритм решения. При этом учащиеся могут попрактиковаться в выполнении задач различного уровня сложности. В случае необходимости упражнение можно сохранить в разделе «Избранное», чтобы потом обсудить решение с преподавателем.

    Вам понадобится

    • — математический справочник;
    • — тетрадь;
    • — простой карандаш;
    • — ручка;
    • — транспортир;
    • — циркуль.

    Инструкция

    Примите к сведению, что график дифференцируемой функции f(x) в точке х0 не имеет различий с отрезком касательной. Поэтому он достаточно близким к отрезку l, к проходящему через точки (х0; f(х0)) и (х0+Δx; f(x0 + Δx)). Чтобы задать прямую, проходящую через точку А с коэффициентами (х0; f(х0)), укажите ее угловой коэффициент. При этом он равен Δy/Δx секущей касательной (Δх→0) , а также стремится к числу f‘(x0).

    Если значений f‘(x0) не существует, то, касательной нет, или же она проходит вертикально. Исходя из этого, производной функции в точке х0 объясняется существованием невертикальной касательной, которая соприкасается с графиком функции в точке (х0, f(х0)). В данном случае угловой коэффициент касательной равняется f»(х0). Становится понятен геометрический производной, то есть углового коэффициента касательной.

    То есть для того чтобы найти угловой коэффициент касательной, нужно найти значение производной функции в точке касания. Пример: найти угловой коэффициент касательной к функции у = х³ в точке с абсциссой Х0 = 1. Решение: Найдите производную данной функции у΄(х) = 3х²; найдите значение производной в точке Х0 = 1. у΄(1) = 3 × 1² = 3. Угловой коэффициент касательной в точке Х0 = 3.

    Начертите на рисунке дополнительные касательные таким образом, чтобы они соприкасались с графиком функции в точках: x1, х2 и х3. Отметьте углы, которые образуются данными касательными с осью абсцисс (угол отсчитывается в положительном направлении — от оси до касательной прямой). Например, угол α1 будет острым, же (α2) – тупой, ну а третий (α3) будет равняться нулю, так как проведенная касательная прямая является параллельной оси ОХ. В этом случае тангенс тупого угла есть отрицательное значение, а тангенс острого угла – положительное, при tg0 и результат равен нулю.

    Касательной к заданной окружности называется прямая линяя, которая имеет только одну общую точку с этой окружностью. Касательная к окружности всегда перпендикулярна его радиусу, проведённому к точке касания. Если две касательные проведены из одной точки, не принадлежащей окружности, то расстояния от этой точки до точек касания всегда будет одинаковым. Касательные к окружностям строятся разными способами, в зависимости от их расположения относительно друг друга.

    Инструкция

    Построение касательной к одной окружности.
    1. Строится окружность радиуса R и берётся A, которую будет проходить касательная.
    2. Строится окружность с центром в середине отрезка OA и радиусам равным этого отрезка.
    3. Пересечения двух точками касания касательных проведённых через точку A к заданной окружности.

    Внешняя касательная к двум окружностям .

    2. Проводится окружность радиусом R – r с центром в точке O.
    3. К полученной окружности проводится касательная из O1, точка касания обозначена M.
    4. Радиус R проходящий через точку M на точку T – точку касания окружности.
    5. Через центр O1 малой окружности проводится радиус r параллельно R большой окружности. Радиус r указывает на точку T1 – точку касания малой окружности.
    окружностям .

    Внутренняя касательная к двум окружностям .
    1. Строятся две окружности радиусом R и r.
    2. Проводится окружность радиусом R + r с центром в точке O.
    3. К полученной окружности проводится касательная из точки O1, точка касания обозначена буквой M.
    4. Луч OM пересекает первую окружность в точке T – в точке касания большой окружности.
    5. Через центр O1 малой окружности проводится радиус r параллельно лучу OM. Радиус r указывает на точку T1 – точку касания малой окружности.
    6. Прямая TT1 – касательная к заданным окружностям .

    Источники:

    • внутренняя касательная

    Угловой шкаф – идеальный вариант для пустующих углов в квартире. Кроме того, конфигурация угловых шкаф ов придает интерьеру классическую атмосферу. В качестве отделки угловых шкаф ов может быть использован любой материал, который подходит для этой цели.

    Вам понадобится

    • ДВП, МДФ, шурупы, гвозди, пильный диск, фриз.

    Инструкция

    Вырежьте из фанеры или ДВП шаблон шириной 125 мм, длиной 1065 мм. Кромки необходимо запилить под углом 45 градусов. По готовому шаблону определите размеры боковых стенок, а так же место, где будет расположен шкаф .

    Крышку соедините с боковыми стенками и треугольными полками. Крепление крышки должно происходить к верхним кромкам боковых стенок при помощи шурупов. Для прочности конструкции дополнительно используют клей. Полки прикрепите к планкам.

    Наклоните пильный диск под углом 45 градусов и скосите по направляющей планке переднюю кромку боковых стенок. Неподвижные полки прикрепите к планкам МДФ. Соедините боковые стенки при помощи шурупов. Следите за тем, чтобы не было щелей.

    В стене сделайте отметки, между которыми поставьте каркас углового шкаф а. С помощью шурупов прикрепите шкаф к стене. Длина дюбеля должна быть 75 мм.

    Из цельной плиты МДФ выпилите лицевую рамку. С помощью дисковой пилы вырежьте в ней проемы, используя линейку. Допилите углы.

    Найдите значение абсциссы точки касания, которую обозначаются буквой «а». Если она совпадает с заданной точкой касательной, то «а» будет ее х-координате. Определите значение функции f(a), подставив в уравнение функции величину абсциссы.

    Определите первую производную уравнения функции f’(x) и подставьте в него значение точки «а».

    Возьмите общее уравнение касательной, которое определяется как y = f(a) = f (a)(x – a), и подставьте в него найденные значения a, f(a), f «(a). В результате будет найдено решение графика и касательной.

    Решите задачу иным способом, если заданная точка касательной не совпала с точкой касания. В этом случае необходимо в уравнение касательной вместо цифр подставить «а». После этого вместо букв «х» и «у» подставьте значение координат заданной точки. Решите получившееся уравнение, в котором «а» является неизвестной. Поставьте полученное значение в уравнение касательной.

    Составьте уравнение касательной с буквой «а», если в условии задачи задано уравнение функции и уравнение параллельной линии относительно искомой касательной. После этого необходимо производную функции , чтобы координату у точки «а». Подставьте соответствующее значение в уравнение касательной и решите функцию.

    При составлении уравнения касательной к графику функции используется понятие «абсцисса точки касания». Данная величина может задаваться изначально в условиях задачи или же ее необходимо определять самостоятельно.

    Инструкция

    Начертите на листе в клеточку оси координат х и у. Изучите заданное уравнение для графика функции. Если оно является , то достаточно два значения для параметра у при любых х, после чего построить найденные точки на оси координат и соединить их линией. Если же график нелинейный, то составьте таблицу зависимости у от х и подберите как минимум пять точек для построения графика.

    Определите значение абсциссы точки касания для случая, когда заданная точка касательной не совпадает с графиком функции. Задаем третий параметр буквой «а».

    Запишите уравнение функции f(a). Для этого в исходное уравнение вместо х подставьте а. Найдите производную функции f(x) и f(a). Подставьте необходимые данные в общее уравнение касательной, которое имеет вид: y = f(a) + f «(a)(x – a). В результате получить уравнение, которое из трех неизвестных параметров.

    Подставьте в него вместо х и у координаты заданной точки, через которую проходит касательная. После этого найдите решение полученного уравнения для всех а. Если оно является квадратным, то будет два значения абсциссы точки касания. Это , что касательная проходит два раза возле графика функции.

    Нарисуйте график заданной функции и , которые заданы по условию задачи. В этом случае необходимо также задать неизвестный параметр а и подставить его в уравнение f(a). Приравняйте производную f(a) к производной уравнения параллельной прямой. Данное выходит из условия параллельности двух . Найдите корни полученного уравнения, которые будут являться абсциссами точки касания.

    Прямая y=f(x) будет касательной к изображенному на рисунке графику в точке х0 в том случае, если она проходит через точку с координатами (х0; f(x0)) и обладает угловым коэффициентом f»(x0). Найти такой коэффициент, зная особенности касательной, несложно.

    Вам понадобится

    • — математический справочник;
    • — простой карандаш;
    • — тетрадь;
    • — транспортир;
    • — циркуль;
    • — ручка.

    Инструкция

    Если значения f‘(x0) не существует, то либо касательной нет, либо она проходит вертикально. Ввиду этого, наличие производной функции в точке х0 обусловлено существованием невертикальной касательной, соприкасающейся с графиком функции в точке (х0, f(х0)). В этом случае угловой коэффициент касательной равен будет f»(х0). Таким образом, становится ясен геометрический смысл производной – расчет углового коэффициента касательной.

    Определите общую . Подобного рода сведения можно получить, обратившись к данным переписи населения. Для определения общих коэффициентов рождаемости, смертности, брачности и разводимости вам понадобится найти произведение общей населения и расчетного периода. Получившееся число запишите в знаменатель.

    Поставьте на числителя показатель, соответствующий искомому относительному. Например, если перед вами стоит определить общий коэффициент рождаемости, то на месте числителя должно находиться число, отражающее общее количество рожденных за интересующий вас период. Если вашей целью является уровня смертности или брачности, то на место числителя поставьте число умерших в расчетный период или число вступивших в брак, соответственно.

    Умножьте получившееся число на 1000. Это и будет искомый вами общий коэффициент. Если же перед вами стоит задача найти общий коэффициент прироста, то вычтите из коэффициента рождаемости коэффициент смертности.

    Видео по теме

    Источники:

    Главным показателем эффективности экстракции является коэффициент распределения . Он считается по формуле: Со/Св, где Со – концентрация извлекаемого вещества в органическом растворителе (экстракторе), а Св – концентрация этого же вещества в воде, после наступления равновесия. Как можно опытным путем найти коэффициент распределения?

    Соблюдение Вашей конфиденциальности важно для нас. По этой причине, мы разработали Политику Конфиденциальности, которая описывает, как мы используем и храним Вашу информацию. Пожалуйста, ознакомьтесь с нашими правилами соблюдения конфиденциальности и сообщите нам, если у вас возникнут какие-либо вопросы.

    Сбор и использование персональной информации

    Под персональной информацией понимаются данные, которые могут быть использованы для идентификации определенного лица либо связи с ним.

    От вас может быть запрошено предоставление вашей персональной информации в любой момент, когда вы связываетесь с нами.

    Ниже приведены некоторые примеры типов персональной информации, которую мы можем собирать, и как мы можем использовать такую информацию.

    Какую персональную информацию мы собираем:

    • Когда вы оставляете заявку на сайте, мы можем собирать различную информацию, включая ваши имя, номер телефона, адрес электронной почты и т.д.

    Как мы используем вашу персональную информацию:

    • Собираемая нами персональная информация позволяет нам связываться с вами и сообщать об уникальных предложениях, акциях и других мероприятиях и ближайших событиях.
    • Время от времени, мы можем использовать вашу персональную информацию для отправки важных уведомлений и сообщений.
    • Мы также можем использовать персональную информацию для внутренних целей, таких как проведения аудита, анализа данных и различных исследований в целях улучшения услуг предоставляемых нами и предоставления Вам рекомендаций относительно наших услуг.
    • Если вы принимаете участие в розыгрыше призов, конкурсе или сходном стимулирующем мероприятии, мы можем использовать предоставляемую вами информацию для управления такими программами.

    Раскрытие информации третьим лицам

    Мы не раскрываем полученную от Вас информацию третьим лицам.

    Исключения:

    • В случае если необходимо — в соответствии с законом, судебным порядком, в судебном разбирательстве, и/или на основании публичных запросов или запросов от государственных органов на территории РФ — раскрыть вашу персональную информацию. Мы также можем раскрывать информацию о вас если мы определим, что такое раскрытие необходимо или уместно в целях безопасности, поддержания правопорядка, или иных общественно важных случаях.
    • В случае реорганизации, слияния или продажи мы можем передать собираемую нами персональную информацию соответствующему третьему лицу – правопреемнику.

    Защита персональной информации

    Мы предпринимаем меры предосторожности — включая административные, технические и физические — для защиты вашей персональной информации от утраты, кражи, и недобросовестного использования, а также от несанкционированного доступа, раскрытия, изменения и уничтожения.

    Соблюдение вашей конфиденциальности на уровне компании

    Для того чтобы убедиться, что ваша персональная информация находится в безопасности, мы доводим нормы соблюдения конфиденциальности и безопасности до наших сотрудников, и строго следим за исполнением мер соблюдения конфиденциальности.

    Угловой коэффициент касательной к графику функции совпадает. Как найти угловой коэффициент

    Прямая y = f(x) будет являться касательной к графику, изображенному на рисунке в точке х0 при том условии, если она проходит через данную точку с координатами (х0; f(x0)) и имеет угловой коэффициент f»(x0). Найти этот коэффициент, учитывая особенности касательной, несложно.

    Вам понадобится

    • — математический справочник;
    • — тетрадь;
    • — простой карандаш;
    • — ручка;
    • — транспортир;
    • — циркуль.

    Инструкция

    • Примите к сведению, что график дифференцируемой функции f(x) в точке х0 не имеет различий с отрезком касательной. Поэтому он является достаточно близким к отрезку l, к проходящему через точки (х0; f(х0)) и (х0+Δx; f(x0 + Δx)). Чтобы задать прямую, проходящую через точку А с коэффициентами (х0; f(х0)), укажите ее угловой коэффициент. При этом он равен Δy/Δx секущей касательной (Δх→0) , а также стремится к числу f‘(x0).
    • Если значений f‘(x0) не существует, то, возможно, касательной нет, или же она проходит вертикально. Исходя из этого, присутствие производной функции в точке х0 объясняется существованием невертикальной касательной, которая соприкасается с графиком функции в точке (х0, f(х0)). В данном случае угловой коэффициент касательной равняется f»(х0). Становится понятен геометрический смысл производной, то есть расчет углового коэффициента касательной.
    • То есть для того чтобы найти угловой коэффициент касательной, нужно найти значение производной функции в точке касания. Пример: найти угловой коэффициент касательной к графику функции у = х³ в точке с абсциссой Х0 = 1. Решение: Найдите производную данной функции у΄(х) = 3х²; найдите значение производной в точке Х0 = 1. у΄(1) = 3 × 1² = 3. Угловой коэффициент касательной в точке Х0 = 1 равен 3.
    • Начертите на рисунке дополнительные касательные таким образом, чтобы они соприкасались с графиком функции в следующих точках: x1, х2 и х3. Отметьте углы, которые образуются данными касательными с осью абсцисс (угол отсчитывается в положительном направлении — от оси до касательной прямой). Например, первый угол α1 будет острым, второй же (α2) – тупой, ну а третий (α3) будет равняться нулю, так как проведенная касательная прямая является параллельной оси ОХ. В этом случае тангенс тупого угла есть отрицательное значение, а тангенс острого угла – положительное, при tg0 и результат равен нулю.

    Теме «Угловой коэффициент касательной как тангенс угла наклона» в аттестационном экзамене отводится сразу несколько заданий. В зависимости от их условия, от выпускника может требоваться как полный ответ, так и краткий. При подготовке к сдаче ЕГЭ по математике ученику обязательно стоит повторить задачи, в которых требуется вычислить угловой коэффициент касательной.

    Сделать это вам поможет образовательный портал «Школково». Наши специалисты подготовили и представили теоретический и практический материал максимально доступно. Ознакомившись с ним, выпускники с любым уровнем подготовки смогут успешно решать задачи, связанные с производными, в которых требуется найти тангенс угла наклона касательной.

    Основные моменты

    Для нахождения правильного и рационального решения подобных заданий в ЕГЭ необходимо вспомнить базовое определение: производная представляет собой скорость изменения функции; она равна тангенсу угла наклона касательной, проведенной к графику функции в определенной точке. Не менее важно выполнить чертеж. Он позволит найти правильное решение задач ЕГЭ на производную, в которых требуется вычислить тангенс угла наклона касательной. Для наглядности лучше всего выполнить построение графика на плоскости ОХY.

    Если вы уже ознакомились с базовым материалом на тему производной и готовы приступить к решению задач на вычисление тангенса угла наклона касательной, подобных заданиям ЕГЭ, сделать это можно в режиме онлайн. Для каждого задания, например, задач на тему «Связь производной со скоростью и ускорением тела» , мы прописали правильный ответ и алгоритм решения. При этом учащиеся могут попрактиковаться в выполнении задач различного уровня сложности. В случае необходимости упражнение можно сохранить в разделе «Избранное», чтобы потом обсудить решение с преподавателем.

    В математике одним из параметров, описывающих положение прямой на декартовой плоскости координат, является угловой коэффициент этой прямой. Этот параметр характеризует наклон прямой к оси абцисс. Чтобы понять, как найти угловой коэффициент, сначала вспомним общий вид уравнения прямой в системе координат XY.

    В общем виде любую прямую можно представить выражением ax+by=c, где a, b и c — произвольные действительные числа, но обязательно a 2 + b 2 ≠ 0.

    Подобное уравнение с помощью несложных преобразований можно довести до вида y=kx+d, в котором k и d — действительные числа. Число k является угловым коэффициентом, а само уравнение прямой подобного вида называется уравнением с угловым коэффициентом. Получается, что для нахождения углового коэффициента, необходимо просто привести исходное уравнение к указанному выше виду. Для более полного понимания рассмотрим конкретный пример:

    Задача: Найти угловой коэффициент линии, заданной уравнением 36x — 18y = 108

    Решение: Преобразуем исходное уравнение.

    Ответ: Искомый угловой коэффициент данной прямой равен 2.

    В случае, если в ходе преобразований уравнения мы получили выражение типа x = const и не можем в результате представить y в виде функции x, то мы имеем дело с прямой, параллельной оси Х. Угловой коэффициент подобной прямой равен бесконечности.

    Для прямых, которых выражены уравнением типа y = const, угловой коэффициент равняется нулю. Это характерно для прямых, параллельных оси абцисс. Например:

    Задача: Найти угловой коэффициент линии, заданной уравнением 24x + 12y — 4(3y + 7) = 4

    Решение: Приведем исходное уравнение к общему виду

    24x + 12y — 12y + 28 = 4

    Из полученного выражения выразить y невозможно, следовательно угловой коэффициент данной прямой равен бесконечности, а сама прямая будет параллельна оси Y.

    Геометрический смысл

    Для лучшего понимания обратимся к картинке:

    На рисунке мы видим график функции типа y = kx. Для упрощения примем коэффициент с = 0. В треугольнике ОАВ отношение стороны ВА к АО будет равно угловому коэффициенту k. Вместе с тем отношение ВА/АО — это тангенс острого угла α в прямоугольном треугольнике ОАВ. Получается, что угловой коэффициент прямой равняется тангенсу угла, который составляет эта прямая с осью абцисс координатной сетки.

    Решая задачу, как найти угловой коэффициент прямой, мы находим тангенс угла между ней и осью Х сетки координат. Граничные случаи, когда рассматриваемая прямая параллельна осям координат, подтверждают вышенаписанное. Действительно для прямой, описанной уравнением y=const, угол между ней и осью абцисс равен нулю. Тангенс нулевого угла также равен нулю и угловой коэффициент тоже равен нулю.

    Для прямых, перпендикулярных оси абцисс и описываемых уравнением х=const, угол между ними и осью Х равен 90 градусов. Тангенс прямого угла равен бесконечности, так же и угловой коэффициент подобных прямых равен бесконечности, что подтверждает написанное выше.

    Угловой коэффициент касательной

    Распространенной, часто встречающейся на практике, задачей является также нахождение углового коэффициента касательной к графику функции в некоторой точке. Касательная — это прямая, следовательно к ней также применимо понятие углового коэффициента.

    Чтобы разобраться, как найти угловой коэффициент касательной, нам будет необходимо вспомнить понятие производной. Производная от любой функции в некоторой точке — это константа, численно равная тангенсу угла, который образуется между касательной в указанной точке к графику этой функции и осью абцисс. Получается, что для определения углового коэффициента касательной в точке x 0 , нам необходимо рассчитать значение производной исходной функции в этой точке k = f»(x 0). Рассмотрим на примере:

    Задача: Найти угловой коэффициент линии, касательной к функции y = 12x 2 + 2xe x при х = 0,1.

    Решение: Найдем производную от исходной функции в общем виде

    y»(0,1) = 24 . 0,1 + 2 . 0,1 . e 0,1 + 2 . e 0,1

    Ответ: Искомый угловой коэффициент в точке х = 0,1 равен 4,831

    Рассмотрим следующий рисунок:

    На нем изображена некоторая функция y = f(x), которая дифференцируема в точке a. Отмечена точка М с координатами (а; f(a)). Через произвольную точку Р(a + ∆x; f(a + ∆x)) графика проведена секущая МР.

    Если теперь точку Р сдвигать по графику к точке М, то прямая МР будет поворачиваться вокруг точки М. При этом ∆х будет стремиться к нулю. Отсюда можно сформулировать определение касательной к графику функции.

    Касательная к графику функции

    Касательная к графику функции есть предельное положение секущей при стремлении приращения аргумента к нулю. Следует понимать, что существование производной функции f в точке х0, означает, что в этой точке графика существует касательная к нему.

    При этом угловой коэффициент касательной будет равен производной этой функции в этой точке f’(x0). В этом заключается геометрический смысл производной. Касательная к графику дифференцируемой в точке х0 функции f — это некоторая прямая, проходящая через точку (x0;f(x0)) и имеющая угловой коэффициент f’(x0).

    Уравнение касательной

    Попытаемся получить уравнение касательной к графику некоторой функции f в точке А(x0; f(x0)). Уравнение прямой с угловым коэффициентом k имеет следующий вид:

    Так как у нас угловой коэффициент равен производной f’(x0) , то уравнение примет следующий вид: y = f’(x0) *x + b.

    Теперь вычислим значение b. Для этого используем тот факт, что функция проходит через точку А.

    f(x0) = f’(x0)*x0 + b, отсюда выражаем b и получим b = f(x0) — f’(x0)*x0.

    Подставляем полученное значение в уравнение касательной:

    y = f’(x0)*x + b = f’(x0)*x + f(x0) — f’(x0)*x0 = f(x0) + f’(x0)*(x — x0).

    y = f(x0) + f’(x0)*(x — x0).

    Рассмотрим следующий пример: найти уравнение касательной к графику функции f(x) = x 3 — 2*x 2 + 1 в точке х = 2.

    2. f(x0) = f(2) = 2 2 — 2*2 2 + 1 = 1.

    3. f’(x) = 3*x 2 — 4*x.

    4. f’(x0) = f’(2) = 3*2 2 — 4*2 = 4.

    5. Подставим полученные значения в формулу касательной, получим: y = 1 + 4*(x — 2). Раскрыв скобки и приведя подобные слагаемые получим: y = 4*x — 7.

    Ответ: y = 4*x — 7.

    Общая схема составления уравнения касательной к графику функции y = f(x):

    1. Определить х0.

    2. Вычислить f(x0).

    3. Вычислить f’(x)

    Как найти угловой коэффициент прямой по графику.

    Уравнение касательной к графику функции. Исчерпывающий гид (2019)

    Научитесь брать производные от функций. Производная характеризует скорость изменения функции в определенной точке, лежащей на графике этой функции. В данном случае графиком может быть как прямая, так и кривая линия. То есть производная характеризует скорость изменения функции в конкретный момент времени. Вспомните общие правила, по которым берутся производные, и только потом переходите к следующему шагу.

    • Прочитайте статью .
    • Как брать простейшие производные, например, производную показательного уравнения, описано . Вычисления, представленные в следующих шагах, будут основаны на описанных в ней методах.

    Научитесь различать задачи, в которых угловой коэффициент требуется вычислить через производную функции. В задачах не всегда предлагается найти угловой коэффициент или производную функции. Например, вас могут попросить найти скорость изменения функции в точке А(х,у). {2}+6x} в точке А(4,2) равен 22.

  • Если возможно, проверьте полученный ответ на графике. Помните, что угловой коэффициент можно вычислить не в каждой точке. Дифференциальное исчисление рассматривает сложные функции и сложные графики, где угловой коэффициент можно вычислить не в каждой точке, а в некоторых случаях точки вообще не лежат на графиках. Если возможно, используйте графический калькулятор, чтобы проверить правильность вычисления углового коэффициента данной вам функции. В противном случае проведите касательную к графику в данной вам точке и подумайте, соответствует ли найденное вами значение углового коэффициента тому, что вы видите на графике.

    • Касательная будет иметь тот же угловой коэффициент, что и график функции в определенной точке. Для того, чтобы провести касательную в данной точке, двигайтесь вправо/влево по оси Х (в нашем примере на 22 значения вправо), а затем вверх на единицу по оси Y. Отметьте точку, а затем соедините ее с данной вам точкой. В нашем примере соедините точки с координатами (4,2) и (26,3).
  • Угловой коэффициент прямой. В этой статье мы с вами рассмотрим задачи связанные с координатной плоскостью включённые в ЕГЭ по математике. Это задания на:

    — определение углового коэффициента прямой, когда известны две точки через которые она проходит;
    — определение абсциссы или ординаты точки пересечения двух прямых на плоскости.

    Что такое абсцисса и ордината точки было описано в данной рубрики. В ней мы уже рассмотрели несколько задач связанных с координатной плоскостью. Что необходимо понимать для рассматриваемого типа задач? Немного теории.

    Уравнение прямой на координатной плоскости имеет вид:

    где k это и есть угловой коэффициент прямой.

    Следующий момент! Угловой коэффициент прямой равен тангенсу угла наклона прямой. Это угол между данной прямой и осью ох.



    Он лежит в пределах от 0 до 180 градусов.

    То есть, если мы приведём уравнение прямой к виду y = kx + b , то далее всегда сможем определить коэффициент k (угловой коэффициент).

    Так же, если мы исходя из условия сможем определить тангенс угла наклона прямой, то тем самым найдём её угловой коэффициент.

    Следующий теоретический момент! Уравнение прямой походящей через две данные точки. Формула имеет вид:


    Рассмотрим задачи (аналогичные задачам из открытого банка заданий):

    Найдите угловой коэффициент прямой, проходящей через точки с координатами (–6;0) и (0;6).


    В данной задаче самый рациональный путь решения это найти тангенс угла между осью ох и данной прямой. Известно, что он равен угловому коэффициенту. Рассмотрим прямоугольный треугольник образованный прямой и осями ох и оу:


    Тангенсом угла в прямоугольном треугольнике является отношение противолежащего катета к прилежащему:

    *Оба катета равны шести (это их длины).

    Конечно, данную задачу можно решить используя формулу нахождения уравнения прямой проходящей через две данные точки. Но это будет более длительный путь решения.

    Ответ: 1

    Найдите угловой коэффициент прямой, проходящей через точки с координатами (5;0) и (0;5).


    Наши точки имеют координаты (5;0) и (0;5). Значит,

    Приведём формулу к виду y = kx + b

    Получили, что угловой коэффициент k = – 1.

    Ответ: –1

    Прямая a проходит через точки с координатами (0;6) и (8;0). Прямая b проходит через точку с координатами (0;10) и параллельна прямой a b с осью оx.


    В данной задаче можно найти уравнение прямой a , определить угловой коэффициент для неё. У прямой b угловой коэффициент будет такой же, так как они параллельны. Далее можно найти уравнение прямой b . А затем, подставив в него значение y = 0, найти абсциссу. НО!

    В данном случае, проще использовать свойство подобия треугольников.

    Прямоугольные треугольники, образованные данными (параллельными) прямыми о осями координат подобны, а это значит, что отношения их соответствующих сторон равны.


    Искомая абсцисса равна 40/3.

    Ответ: 40/3

    Прямая a проходит через точки с координатами (0;8) и (–12;0). Прямая b проходит через точку с координатами (0; –12) и параллельна прямой a . Найдите абсциссу точки пересечения прямой b с осью оx .


    Для данной задачи самый рациональный путь решения — это применение свойства подобия треугольников. Но мы решим её другим путём.

    Нам известны точки, через которые проходит прямая а . Можем составить уравнение прямой. Формула уравнения прямой походящей через две данные точки имеет вид:


    По условию точки имеют координаты (0;8) и (–12;0). Значит,

    Приведём к виду y = kx + b :

    Получили, что угловой k = 2/3.

    *Угловой коэффициент можно было найти через тангенс угла в прямоугольном треугольнике с катетами 8 и 12.

    Известно, у параллельных прямых угловые коэффициенты равны. Значит уравнение прямой проходящей через точку (0;-12) имеет вид:

    Найти величину b мы можем подставив абсциссу и ординату в уравнение:

    Таким образом, прямая имеет вид:

    Теперь чтобы найти искомую абсциссу точки пересечения прямой с осью ох, необходимо подставить у = 0:

    Ответ: 18

    Найдите ординату точки пересечения оси оy и прямой, проходящей через точку В(10;12) и параллельной прямой, проходящей через начало координат и точку А(10;24).


    Найдём уравнение прямой проходящей через точки с координатами (0;0) и (10;24).

    Формула уравнения прямой походящей через две данные точки имеет вид:

    Наши точки имеют координаты (0;0) и (10;24). Значит,

    Приведём к виду y = kx + b

    Угловые коэффициенты параллельных прямых равны. Значит, уравнение прямой, проходящей через точку В(10;12) имеет вид:

    Значение b найдём подставив в это уравнение координаты точки В(10;12):

    Получили уравнение прямой:

    Чтобы найти ординату точки пересечения этой прямой с осью оу нужно подставить в найденное уравнение х = 0:

    *Самый простой способ решения. При помощи параллельного переноса сдвигаем данную прямую вниз вдоль оси оу до точки (10;12). Сдвиг происходит на 12 единиц, то есть точка А(10;24) «перешла» в точку В(10;12), а точка О(0;0) «перешла» в точку (0;–12). Значит, полученная прямая будет пересекать ось оу в точке (0;–12).

    Искомая ордината равна –12.

    Ответ: –12

    Найдите ординату точки пересечения прямой, заданной уравнением

    + 2у = 6 , с осью Oy .

    Координата точки пересечения заданной прямой с осью оу имеет вид (0;у ). Подставим в уравнение абсциссу х = 0, и найдём ординату:

    Ордината точки пересечения прямой с осью оу равна 3.

    *Решается система:

    Ответ: 3

    Найдите ординату точки пересечения прямых, заданных уравнениями

    3х + 2у = 6 и у = – х .

    Когда заданны две прямые, и стоит вопрос о нахождении координат точки пересечения этих прямых, решается система из данных уравнений:

    В первом уравнении подставляем – х вместо у :

    Ордината равна минус шести.

    Ответ: 6

    Найдите угловой коэффициент прямой, проходящей через точки с координатами (–2;0) и (0;2).

    Найдите угловой коэффициент прямой, проходящей через точки с координатами (2;0) и (0;2).

    Прямая a проходит через точки с координатами (0;4) и (6;0). Прямая b проходит через точку с координатами (0;8) и параллельна прямой a. Найдите абсциссу точки пересечения прямой b с осью Ox.

    Найдите ординату точки пересечения оси оy и прямой, проходящей через точку B (6;4) и параллельной прямой, проходящей через начало координат и точку A (6;8).

    1. Необходимо чётко усвоить, что угловой коэффициент прямой равен тангенсу угла наклона прямой. Это поможет вам при решении многих задач данного типа.

    2. Формулу нахождения прямой проходящей через две данные точки нужно понимать обязательно. С её помощью всегда найдёте уравнение прямой, если даны координаты двух её точек.

    3. Помните о том, что угловые коэффициенты параллельных прямых равны.

    4. Как вы поняли, в некоторых задачах удобно использовать признак подобия треугольников. Задачи решаются практически устно.

    5. Задачи в которых даны две прямые и требуется найти абсциссу или ординату точки их пересечения можно решить графическим способом. То есть, построить их на координатной плоскости (на листе в клетку) и определить точку пересечения визуально. *Но этот способ применим не всегда.

    6. И последнее. Если дана прямая и координаты точек её пересечения с осями координат, то в таких задачах удобно находить угловой коэффициент через нахождение тангенса угла в образованном прямоугольном треугольнике. Как «увидеть» этот треугольник при различных расположениях прямых на плоскости схематично показано ниже:

    >> Угол наклона прямой от 0 до 90 градусов


    >> Угол наклона прямой от 90 до 180 градусов

    На этом всё. Успеха Вам!

    С уважением, Александр.

    P.S: Буду благодарен Вам, если расскажете о сайте в социальных сетях.

    В математике одним из параметров, описывающих положение прямой на декартовой плоскости координат, является угловой коэффициент этой прямой. Этот параметр характеризует наклон прямой к оси абцисс. Чтобы понять, как найти угловой коэффициент, сначала вспомним общий вид уравнения прямой в системе координат XY.

    В общем виде любую прямую можно представить выражением ax+by=c, где a, b и c — произвольные действительные числа, но обязательно a 2 + b 2 ≠ 0.

    Подобное уравнение с помощью несложных преобразований можно довести до вида y=kx+d, в котором k и d — действительные числа. Число k является угловым коэффициентом, а само уравнение прямой подобного вида называется уравнением с угловым коэффициентом. Получается, что для нахождения углового коэффициента, необходимо просто привести исходное уравнение к указанному выше виду. Для более полного понимания рассмотрим конкретный пример:

    Задача: Найти угловой коэффициент линии, заданной уравнением 36x — 18y = 108

    Решение: Преобразуем исходное уравнение.

    Ответ: Искомый угловой коэффициент данной прямой равен 2.

    В случае, если в ходе преобразований уравнения мы получили выражение типа x = const и не можем в результате представить y в виде функции x, то мы имеем дело с прямой, параллельной оси Х. Угловой коэффициент подобной прямой равен бесконечности.

    Для прямых, которых выражены уравнением типа y = const, угловой коэффициент равняется нулю. Это характерно для прямых, параллельных оси абцисс. Например:

    Задача: Найти угловой коэффициент линии, заданной уравнением 24x + 12y — 4(3y + 7) = 4

    Решение: Приведем исходное уравнение к общему виду

    24x + 12y — 12y + 28 = 4

    Из полученного выражения выразить y невозможно, следовательно угловой коэффициент данной прямой равен бесконечности, а сама прямая будет параллельна оси Y.

    Геометрический смысл

    Для лучшего понимания обратимся к картинке:

    На рисунке мы видим график функции типа y = kx. Для упрощения примем коэффициент с = 0. В треугольнике ОАВ отношение стороны ВА к АО будет равно угловому коэффициенту k. Вместе с тем отношение ВА/АО — это тангенс острого угла α в прямоугольном треугольнике ОАВ. Получается, что угловой коэффициент прямой равняется тангенсу угла, который составляет эта прямая с осью абцисс координатной сетки.

    Решая задачу, как найти угловой коэффициент прямой, мы находим тангенс угла между ней и осью Х сетки координат. Граничные случаи, когда рассматриваемая прямая параллельна осям координат, подтверждают вышенаписанное. Действительно для прямой, описанной уравнением y=const, угол между ней и осью абцисс равен нулю. Тангенс нулевого угла также равен нулю и угловой коэффициент тоже равен нулю.

    Для прямых, перпендикулярных оси абцисс и описываемых уравнением х=const, угол между ними и осью Х равен 90 градусов. Тангенс прямого угла равен бесконечности, так же и угловой коэффициент подобных прямых равен бесконечности, что подтверждает написанное выше.

    Угловой коэффициент касательной

    Распространенной, часто встречающейся на практике, задачей является также нахождение углового коэффициента касательной к графику функции в некоторой точке. Касательная — это прямая, следовательно к ней также применимо понятие углового коэффициента.

    Чтобы разобраться, как найти угловой коэффициент касательной, нам будет необходимо вспомнить понятие производной. Производная от любой функции в некоторой точке — это константа, численно равная тангенсу угла, который образуется между касательной в указанной точке к графику этой функции и осью абцисс. Получается, что для определения углового коэффициента касательной в точке x 0 , нам необходимо рассчитать значение производной исходной функции в этой точке k = f»(x 0). Рассмотрим на примере:

    Задача: Найти угловой коэффициент линии, касательной к функции y = 12x 2 + 2xe x при х = 0,1.

    Решение: Найдем производную от исходной функции в общем виде

    y»(0,1) = 24 . 0,1 + 2 . 0,1 . e 0,1 + 2 . e 0,1

    Ответ: Искомый угловой коэффициент в точке х = 0,1 равен 4,831

    Br формула уравнения касательной. Уравнение касательной к графику функции

    Теме «Угловой коэффициент касательной как тангенс угла наклона» в аттестационном экзамене отводится сразу несколько заданий. В зависимости от их условия, от выпускника может требоваться как полный ответ, так и краткий. При подготовке к сдаче ЕГЭ по математике ученику обязательно стоит повторить задачи, в которых требуется вычислить угловой коэффициент касательной.

    Сделать это вам поможет образовательный портал «Школково». Наши специалисты подготовили и представили теоретический и практический материал максимально доступно. Ознакомившись с ним, выпускники с любым уровнем подготовки смогут успешно решать задачи, связанные с производными, в которых требуется найти тангенс угла наклона касательной.

    Основные моменты

    Для нахождения правильного и рационального решения подобных заданий в ЕГЭ необходимо вспомнить базовое определение: производная представляет собой скорость изменения функции; она равна тангенсу угла наклона касательной, проведенной к графику функции в определенной точке. Не менее важно выполнить чертеж. Он позволит найти правильное решение задач ЕГЭ на производную, в которых требуется вычислить тангенс угла наклона касательной. Для наглядности лучше всего выполнить построение графика на плоскости ОХY.

    Если вы уже ознакомились с базовым материалом на тему производной и готовы приступить к решению задач на вычисление тангенса угла наклона касательной, подобных заданиям ЕГЭ, сделать это можно в режиме онлайн. Для каждого задания, например, задач на тему «Связь производной со скоростью и ускорением тела» , мы прописали правильный ответ и алгоритм решения. При этом учащиеся могут попрактиковаться в выполнении задач различного уровня сложности. В случае необходимости упражнение можно сохранить в разделе «Избранное», чтобы потом обсудить решение с преподавателем.

    Пример 1. Дана функция f (x ) = 3x 2 + 4x – 5. Напишем уравнение касательной к графику функции f (x ) в точке графика с абсциссой x 0 = 1.

    Решение. Производная функции f (x ) существует для любого x R . Найдем ее:

    = (3x 2 + 4x – 5)′ = 6x + 4.

    Тогда f (x 0) = f (1) = 2; (x 0) = = 10. Уравнение касательной имеет вид:

    y = (x 0) (x x 0) + f (x 0),

    y = 10(x – 1) + 2,

    y = 10x – 8.

    Ответ. y = 10x – 8.

    Пример 2. Дана функция f (x ) = x 3 – 3x 2 + 2x + 5. Напишем уравнение касательной к графику функции f (x ), параллельной прямой y = 2x – 11.

    Решение. Производная функции f (x ) существует для любого x R . Найдем ее:

    = (x 3 – 3x 2 + 2x + 5)′ = 3x 2 – 6x + 2.

    Так как касательная к графику функции f (x ) в точке с абсциссой x 0 параллельна прямой y = 2x – 11, то ее угловой коэффициент равен 2, т. е. (x 0) = 2. Найдем эту абсциссу из условия, что 3x – 6x 0 + 2 = 2. Это равенство справедливо лишь при x 0 = 0 и при x 0 = 2. Так как в том и в другом случае f (x 0) = 5, то прямая y = 2x + b касается графика функции или в точке (0; 5), или в точке (2; 5).

    В первом случае верно числовое равенство 5 = 2×0 + b , откуда b = 5, а во втором случае верно числовое равенство 5 = 2×2 + b , откуда b = 1.

    Итак, существует две касательные y = 2x + 5 и y = 2x + 1 к графику функции f (x ), параллельные прямой y = 2x – 11.

    Ответ. y = 2x + 5, y = 2x + 1.

    Пример 3. Дана функция f (x ) = x 2 – 6x + 7. Напишем уравнение касательной к графику функции f (x ), проходящей через точку A (2; –5).

    Решение. Так как f (2) –5, то точка A не принадлежит графику функции f (x ). Пусть x 0 — абсцисса точки касания.

    Производная функции f (x ) существует для любого x R . Найдем ее:

    = (x 2 – 6x + 1)′ = 2x – 6.

    Тогда f (x 0) = x – 6x 0 + 7; (x 0) = 2x 0 – 6. Уравнение касательной имеет вид:

    y = (2x 0 – 6)(x x 0) + x – 6x + 7,

    y = (2x 0 – 6)x x + 7.

    Так как точка A принадлежит касательной, то справедливо числовое равенство

    –5 = (2x 0 – 6)×2– x + 7,

    откуда x 0 = 0 или x 0 = 4. Это означает, что через точку A можно провести две касательные к графику функции f (x ).

    Если x 0 = 0, то уравнение касательной имеет вид y = –6x + 7. Если x 0 = 4, то уравнение касательной имеет вид y = 2x – 9.

    Ответ. y = –6x + 7, y = 2x – 9.

    Пример 4. Даны функции f (x ) = x 2 – 2x + 2 и g (x ) = –x 2 – 3. Напишем уравнение общей касательной к графикам этих функции.

    Решение. Пусть x 1 — абсцисса точки касания искомой прямой с графиком функции f (x ), а x 2 — абсцисса точки касания той же прямой с графиком функции g (x ).

    Производная функции f (x ) существует для любого x R . Найдем ее:

    = (x 2 – 2x + 2)′ = 2x – 2.

    Тогда f (x 1) = x – 2x 1 + 2; (x 1) = 2 x 1 – 2. Уравнение касательной имеет вид:

    y = (2x 1 – 2)(x x 1) + x – 2x 1 + 2,

    y = (2x 1 – 2)x x + 2. (1)

    Найдем производную функции g (x ):

    = (–x 2 – 3)′ = –2x .

    Статья дает подробное разъяснение определений, геометрического смысла производной с графическими обозначениями. Будет рассмотрено уравнение касательной прямой с приведением примеров, найдено уравнения касательной к кривым 2 порядка.

    Yandex.RTB R-A-339285-1 Определение 1

    Угол наклона прямой y = k x + b называется угол α , который отсчитывается от положительного направления оси о х к прямой y = k x + b в положительном направлении.

    На рисунке направление о х обозначается при помощи зеленой стрелки и в виде зеленой дуги, а угол наклона при помощи красной дуги. Синяя линия относится к прямой.

    Определение 2

    Угловой коэффициент прямой y = k x + b называют числовым коэффициентом k .

    Угловой коэффициент равняется тангенсу наклона прямой, иначе говоря k = t g α .

    • Угол наклона прямой равняется 0 только при параллельности о х и угловом коэффициенте, равному нулю, потому как тангенс нуля равен 0 . Значит, вид уравнения будет y = b .
    • Если угол наклона прямой y = k x + b острый, тогда выполняются условия 0 0 , причем имеется возрастание графика.
    • Если α = π 2 , тогда расположение прямой перпендикулярно о х. Равенство задается при помощи равенства x = c со значением с, являющимся действительным числом.
    • Если угол наклона прямой y = k x + b тупой, то соответствует условиям π 2
    Определение 3

    Секущей называют прямую, которая проходит через 2 точки функции f (x) . Иначе говоря, секущая – это прямая, которая проводится через любые две точки графика заданной функции.

    По рисунку видно, что А В является секущей, а f (x) – черная кривая, α — красная дуга, означающая угол наклона секущей.

    Когда угловой коэффициент прямой равняется тангенсу угла наклона, то видно, что тангенс из прямоугольного треугольника А В С можно найти по отношению противолежащего катета к прилежащему.

    Определение 4

    Получаем формулу для нахождения секущей вида:

    k = t g α = B C A C = f (x B) — f x A x B — x A , где абсциссами точек А и В являются значения x A , x B , а f (x A) , f (x B) — это значения функции в этих точках.

    Очевидно, что угловой коэффициент секущей определен при помощи равенства k = f (x B) — f (x A) x B — x A или k = f (x A) — f (x B) x A — x B , причем уравнение необходимо записать как y = f (x B) — f (x A) x B — x A · x — x A + f (x A) или
    y = f (x A) — f (x B) x A — x B · x — x B + f (x B) .

    Секущая делит график визуально на 3 части: слева от точки А, от А до В, справа от В. На располагаемом ниже рисунке видно, что имеются три секущие, которые считаются совпадающими, то есть задаются при помощи аналогичного уравнения.

    По определению видно, что прямая и ее секущая в данном случае совпадают.

    Секущая может множественно раз пересекать график заданной функции. Если имеется уравнение вида у = 0 для секущей, тогда количество точек пересечения с синусоидой бесконечно.

    Определение 5

    Касательная к графику функции f (x) в точке x 0 ; f (x 0) называется прямая, проходящая через заданную точку x 0 ; f (x 0) , с наличием отрезка, который имеет множество значений х, близких к x 0 .

    Пример 1

    Рассмотрим подробно на ниже приведенном примере. Тогда видно, что прямая, заданная функцией y = x + 1 , считается касательной к y = 2 x в точке с координатами (1 ; 2) . Для наглядности, необходимо рассмотреть графики с приближенными к (1 ; 2) значениями. Функция y = 2 x обозначена черным цветом, синяя линия – касательная, красная точка – точка пересечения.

    Очевидно, что y = 2 x сливается с прямой у = х + 1 .

    Для определения касательной следует рассмотреть поведение касательной А В при бесконечном приближении точки В к точке А. Для наглядности приведем рисунок.

    Секущая А В, обозначенная при помощи синей линии, стремится к положению самой касательной, а угол наклона секущей α начнет стремиться к углу наклона самой касательной α x .

    Определение 6

    Касательной к графику функции y = f (x) в точке А считается предельное положение секущей А В при В стремящейся к А, то есть B → A .

    Теперь перейдем к рассмотрению геометрического смысла производной функции в точке.

    Перейдем к рассмотрению секущей А В для функции f (x) , где А и В с координатами x 0 , f (x 0) и x 0 + ∆ x , f (x 0 + ∆ x) , а ∆ x обозначаем как приращение аргумента. Теперь функция примет вид ∆ y = ∆ f (x) = f (x 0 + ∆ x) — f (∆ x) . Для наглядности приведем в пример рисунок.

    Рассмотрим полученный прямоугольный треугольник А В С. Используем определение тангенса для решения, то есть получим отношение ∆ y ∆ x = t g α . Из определения касательной следует, что lim ∆ x → 0 ∆ y ∆ x = t g α x . По правилу производной в точке имеем, что производную f (x) в точке x 0 называют пределом отношений приращения функции к приращению аргумента, где ∆ x → 0 , тогда обозначим как f (x 0) = lim ∆ x → 0 ∆ y ∆ x .

    Отсюда следует, что f » (x 0) = lim ∆ x → 0 ∆ y ∆ x = t g α x = k x , где k x обозначают в качестве углового коэффициента касательной.

    То есть получаем, что f ’ (x) может существовать в точке x 0 причем как и касательная к заданному графику функции в точке касания равной x 0 , f 0 (x 0) , где значение углового коэффициента касательной в точке равняется производной в точке x 0 . Тогда получаем, что k x = f » (x 0) .

    Геометрический смысл производной функции в точке в том, что дается понятие существования касательной к графику в этой же точке.

    Чтобы записать уравнение любой прямой на плоскости, необходимо иметь угловой коэффициент с точкой, через которую она проходит. Его обозначение принимается как x 0 при пересечении.

    Уравнение касательной к графику функции y = f (x) в точке x 0 , f 0 (x 0) принимает вид y = f » (x 0) · x — x 0 + f (x 0) .

    Имеется в виду, что конечным значением производной f » (x 0) можно определить положение касательной, то есть вертикально при условии lim x → x 0 + 0 f » (x) = ∞ и lim x → x 0 — 0 f » (x) = ∞ или отсутствие вовсе при условии lim x → x 0 + 0 f » (x) ≠ lim x → x 0 — 0 f » (x) .

    Расположение касательной зависит от значения ее углового коэффициента k x = f » (x 0) . При параллельности к оси о х получаем, что k k = 0 , при параллельности к о у — k x = ∞ , причем вид уравнения касательной x = x 0 возрастает при k x > 0 , убывает при k x

    Пример 2

    Произвести составление уравнения касательной к графику функции y = e x + 1 + x 3 3 — 6 — 3 3 x — 17 — 3 3 в точке с координатами (1 ; 3) с определением угла наклона.

    Решение

    По условию имеем, что функция определяется для всех действительных чисел. Получаем, что точка с координатами, заданными по условию, (1 ; 3) является точкой касания, тогда x 0 = — 1 , f (x 0) = — 3 .

    Необходимо найти производную в точке со значением — 1 . Получаем, что

    y » = e x + 1 + x 3 3 — 6 — 3 3 x — 17 — 3 3 » = = e x + 1 » + x 3 3 » — 6 — 3 3 x » — 17 — 3 3 » = e x + 1 + x 2 — 6 — 3 3 y » (x 0) = y » (- 1) = e — 1 + 1 + — 1 2 — 6 — 3 3 = 3 3

    Значение f ’ (x) в точке касания является угловым коэффициентом касательной, который равняется тангенсу наклона.

    Тогда k x = t g α x = y » (x 0) = 3 3

    Отсюда следует, что α x = a r c t g 3 3 = π 6

    Ответ: уравнение касательной приобретает вид

    y = f » (x 0) · x — x 0 + f (x 0) y = 3 3 (x + 1) — 3 y = 3 3 x — 9 — 3 3

    Для наглядности приведем пример в графической иллюстрации.

    Черный цвет используется для графика исходной функции, синий цвет – изображение касательной, красная точка – точка касания. Рисунок, располагаемый справа, показывает в увеличенном виде.

    Пример 3

    Выяснить наличие существования касательной к графику заданной функции
    y = 3 · x — 1 5 + 1 в точке с координатами (1 ; 1) . Составить уравнение и определить угол наклона.

    Решение

    По условию имеем, что областью определения заданной функции считается множество всех действительных чисел.

    Перейдем к нахождению производной

    y » = 3 · x — 1 5 + 1 » = 3 · 1 5 · (x — 1) 1 5 — 1 = 3 5 · 1 (x — 1) 4 5

    Если x 0 = 1 , тогда f ’ (x) не определена, но пределы записываются как lim x → 1 + 0 3 5 · 1 (x — 1) 4 5 = 3 5 · 1 (+ 0) 4 5 = 3 5 · 1 + 0 = + ∞ и lim x → 1 — 0 3 5 · 1 (x — 1) 4 5 = 3 5 · 1 (- 0) 4 5 = 3 5 · 1 + 0 = + ∞ , что означает существование вертикальной касательной в точке (1 ; 1) .

    Ответ: уравнение примет вид х = 1 , где угол наклона будет равен π 2 .

    Для наглядности изобразим графически.

    Пример 4

    Найти точки графика функции y = 1 15 x + 2 3 — 4 5 x 2 — 16 5 x — 26 5 + 3 x + 2 , где

    1. Касательная не существует;
    2. Касательная располагается параллельно о х;
    3. Касательная параллельна прямой y = 8 5 x + 4 .

    Решение

    Необходимо обратить внимание на область определения. По условию имеем, что функция определена на множестве всех действительных чисел. Раскрываем модуль и решаем систему с промежутками x ∈ — ∞ ; 2 и [ — 2 ; + ∞) . Получаем, что

    y = — 1 15 x 3 + 18 x 2 + 105 x + 176 , x ∈ — ∞ ; — 2 1 15 x 3 — 6 x 2 + 9 x + 12 , x ∈ [ — 2 ; + ∞)

    Необходимо продифференцировать функцию. Имеем, что

    y » = — 1 15 x 3 + 18 x 2 + 105 x + 176 » , x ∈ — ∞ ; — 2 1 15 x 3 — 6 x 2 + 9 x + 12 » , x ∈ [ — 2 ; + ∞) ⇔ y » = — 1 5 (x 2 + 12 x + 35) , x ∈ — ∞ ; — 2 1 5 x 2 — 4 x + 3 , x ∈ [ — 2 ; + ∞)

    Когда х = — 2 , тогда производная не существует, потому что односторонние пределы не равны в этой точке:

    lim x → — 2 — 0 y » (x) = lim x → — 2 — 0 — 1 5 (x 2 + 12 x + 35 = — 1 5 (- 2) 2 + 12 (- 2) + 35 = — 3 lim x → — 2 + 0 y » (x) = lim x → — 2 + 0 1 5 (x 2 — 4 x + 3) = 1 5 — 2 2 — 4 — 2 + 3 = 3

    Вычисляем значение функции в точке х = — 2 , где получаем, что

    1. y (- 2) = 1 15 — 2 + 2 3 — 4 5 (- 2) 2 — 16 5 (- 2) — 26 5 + 3 — 2 + 2 = — 2 , то есть касательная в точке (- 2 ; — 2) не будет существовать.
    2. Касательная параллельна о х, когда угловой коэффициент равняется нулю. Тогда k x = t g α x = f » (x 0) . То есть необходимо найти значения таких х, когда производная функции обращает ее в ноль. То есть значения f ’ (x) и будут являться точками касания, где касательная является параллельной о х.

    Когда x ∈ — ∞ ; — 2 , тогда — 1 5 (x 2 + 12 x + 35) = 0 , а при x ∈ (- 2 ; + ∞) получаем 1 5 (x 2 — 4 x + 3) = 0 .

    1 5 (x 2 + 12 x + 35) = 0 D = 12 2 — 4 · 35 = 144 — 140 = 4 x 1 = — 12 + 4 2 = — 5 ∈ — ∞ ; — 2 x 2 = — 12 — 4 2 = — 7 ∈ — ∞ ; — 2 1 5 (x 2 — 4 x + 3) = 0 D = 4 2 — 4 · 3 = 4 x 3 = 4 — 4 2 = 1 ∈ — 2 ; + ∞ x 4 = 4 + 4 2 = 3 ∈ — 2 ; + ∞

    Вычисляем соответствующие значения функции

    y 1 = y — 5 = 1 15 — 5 + 2 3 — 4 5 — 5 2 — 16 5 — 5 — 26 5 + 3 — 5 + 2 = 8 5 y 2 = y (- 7) = 1 15 — 7 + 2 3 — 4 5 (- 7) 2 — 16 5 — 7 — 26 5 + 3 — 7 + 2 = 4 3 y 3 = y (1) = 1 15 1 + 2 3 — 4 5 · 1 2 — 16 5 · 1 — 26 5 + 3 1 + 2 = 8 5 y 4 = y (3) = 1 15 3 + 2 3 — 4 5 · 3 2 — 16 5 · 3 — 26 5 + 3 3 + 2 = 4 3

    Отсюда — 5 ; 8 5 , — 4 ; 4 3 , 1 ; 8 5 , 3 ; 4 3 считаются искомыми точками графика функции.

    Рассмотрим графическое изображение решения.

    Черная линия – график функции, красные точки – точки касания.

    1. Когда прямые располагаются параллельно, то угловые коэффициенты равны. Тогда необходимо заняться поиском точек графика функции, где угловой коэффициент будет равняться значению 8 5 . Для этого нужно решить уравнение вида y » (x) = 8 5 . Тогда, если x ∈ — ∞ ; — 2 , получаем, что — 1 5 (x 2 + 12 x + 35) = 8 5 , а если x ∈ (- 2 ; + ∞) , тогда 1 5 (x 2 — 4 x + 3) = 8 5 .

    Первое уравнение не имеет корней, так как дискриминант меньше нуля. Запишем, что

    1 5 x 2 + 12 x + 35 = 8 5 x 2 + 12 x + 43 = 0 D = 12 2 — 4 · 43 = — 28

    Другое уравнение имеет два действительных корня, тогда

    1 5 (x 2 — 4 x + 3) = 8 5 x 2 — 4 x — 5 = 0 D = 4 2 — 4 · (- 5) = 36 x 1 = 4 — 36 2 = — 1 ∈ — 2 ; + ∞ x 2 = 4 + 36 2 = 5 ∈ — 2 ; + ∞

    Перейдем к нахождению значений функции. Получаем, что

    y 1 = y (- 1) = 1 15 — 1 + 2 3 — 4 5 (- 1) 2 — 16 5 (- 1) — 26 5 + 3 — 1 + 2 = 4 15 y 2 = y (5) = 1 15 5 + 2 3 — 4 5 · 5 2 — 16 5 · 5 — 26 5 + 3 5 + 2 = 8 3

    Точки со значениями — 1 ; 4 15 , 5 ; 8 3 являются точками, в которых касательные параллельны прямой y = 8 5 x + 4 .

    Ответ: черная линия – график функции, красная линия – график y = 8 5 x + 4 , синяя линия – касательные в точках — 1 ; 4 15 , 5 ; 8 3 .

    Возможно существование бесконечного количества касательных для заданных функций.

    Пример 5

    Написать уравнения всех имеющихся касательных функции y = 3 cos 3 2 x — π 4 — 1 3 , которые располагаются перпендикулярно прямой y = — 2 x + 1 2 .

    Решение

    Для составления уравнения касательной необходимо найти коэффициент и координаты точки касания, исходя из условия перпендикулярности прямых. Определение звучит так: произведение угловых коэффициентов, которые перпендикулярны прямым, равняется — 1 , то есть записывается как k x · k ⊥ = — 1 . Из условия имеем, что угловой коэффициент располагается перпендикулярно прямой и равняется k ⊥ = — 2 , тогда k x = — 1 k ⊥ = — 1 — 2 = 1 2 .

    Теперь необходимо найти координаты точек касания. Нужно найти х, после чего его значение для заданной функции. Отметим, что из геометрического смысла производной в точке
    x 0 получаем, что k x = y » (x 0) . Из данного равенства найдем значения х для точек касания.

    Получаем, что

    y » (x 0) = 3 cos 3 2 x 0 — π 4 — 1 3 » = 3 · — sin 3 2 x 0 — π 4 · 3 2 x 0 — π 4 » = = — 3 · sin 3 2 x 0 — π 4 · 3 2 = — 9 2 · sin 3 2 x 0 — π 4 ⇒ k x = y » (x 0) ⇔ — 9 2 · sin 3 2 x 0 — π 4 = 1 2 ⇒ sin 3 2 x 0 — π 4 = — 1 9

    Это тригонометрическое уравнение будет использовано для вычисления ординат точек касания.

    3 2 x 0 — π 4 = a r c sin — 1 9 + 2 πk или 3 2 x 0 — π 4 = π — a r c sin — 1 9 + 2 πk

    3 2 x 0 — π 4 = — a r c sin 1 9 + 2 πk или 3 2 x 0 — π 4 = π + a r c sin 1 9 + 2 πk

    x 0 = 2 3 π 4 — a r c sin 1 9 + 2 πk или x 0 = 2 3 5 π 4 + a r c sin 1 9 + 2 πk , k ∈ Z

    Z — множество целых чисел.

    Найдены х точек касания. Теперь необходимо перейти к поиску значений у:

    y 0 = 3 cos 3 2 x 0 — π 4 — 1 3

    y 0 = 3 · 1 — sin 2 3 2 x 0 — π 4 — 1 3 или y 0 = 3 · — 1 — sin 2 3 2 x 0 — π 4 — 1 3

    y 0 = 3 · 1 — — 1 9 2 — 1 3 или y 0 = 3 · — 1 — — 1 9 2 — 1 3

    y 0 = 4 5 — 1 3 или y 0 = — 4 5 + 1 3

    Отсюда получаем, что 2 3 π 4 — a r c sin 1 9 + 2 πk ; 4 5 — 1 3 , 2 3 5 π 4 + a r c sin 1 9 + 2 πk ; — 4 5 + 1 3 являются точками касания.

    Ответ: необходимы уравнения запишутся как

    y = 1 2 x — 2 3 π 4 — a r c sin 1 9 + 2 πk + 4 5 — 1 3 , y = 1 2 x — 2 3 5 π 4 + a r c sin 1 9 + 2 πk — 4 5 + 1 3 , k ∈ Z

    Для наглядного изображения рассмотрим функцию и касательную на координатной прямой.

    Рисунок показывает, что расположение функции идет на промежутке [ — 10 ; 10 ] , где черная прямя – график функции, синие линии – касательные, которые располагаются перпендикулярно заданной прямой вида y = — 2 x + 1 2 . Красные точки – это точки касания.

    Канонические уравнения кривых 2 порядка не являются однозначными функциями. Уравнения касательных для них составляются по известным схемам.

    Касательная к окружности

    Для задания окружности с центром в точке x c e n t e r ; y c e n t e r и радиусом R применяется формула x — x c e n t e r 2 + y — y c e n t e r 2 = R 2 .

    Данное равенство может быть записано как объединение двух функций:

    y = R 2 — x — x c e n t e r 2 + y c e n t e r y = — R 2 — x — x c e n t e r 2 + y c e n t e r

    Первая функция располагается вверху, а вторая внизу, как показано на рисунке.

    Для составления уравнения окружности в точке x 0 ; y 0 , которая располагается в верхней или нижней полуокружности, следует найти уравнение графика функции вида y = R 2 — x — x c e n t e r 2 + y c e n t e r или y = — R 2 — x — x c e n t e r 2 + y c e n t e r в указанной точке.

    Когда в точках x c e n t e r ; y c e n t e r + R и x c e n t e r ; y c e n t e r — R касательные могут быть заданы уравнениями y = y c e n t e r + R и y = y c e n t e r — R , а в точках x c e n t e r + R ; y c e n t e r и
    x c e n t e r — R ; y c e n t e r будут являться параллельными о у, тогда получим уравнения вида x = x c e n t e r + R и x = x c e n t e r — R .

    Касательная к эллипсу

    Когда эллипс имеет центр в точке x c e n t e r ; y c e n t e r с полуосями a и b , тогда он может быть задан при помощи уравнения x — x c e n t e r 2 a 2 + y — y c e n t e r 2 b 2 = 1 .

    Эллипс и окружность могут быть обозначаться при помощи объединения двух функций, а именно: верхнего и нижнего полуэллипса. Тогда получаем, что

    y = b a · a 2 — (x — x c e n t e r) 2 + y c e n t e r y = — b a · a 2 — (x — x c e n t e r) 2 + y c e n t e r

    Если касательные располагаются на вершинах эллипса, тогда они параллельны о х или о у. Ниже для наглядности рассмотрим рисунок.

    Пример 6

    Написать уравнение касательной к эллипсу x — 3 2 4 + y — 5 2 25 = 1 в точках со значениями x равного х = 2 .

    Решение

    Необходимо найти точки касания, которые соответствуют значению х = 2 . Производим подстановку в имеющееся уравнение эллипса и получаем, что

    x — 3 2 4 x = 2 + y — 5 2 25 = 1 1 4 + y — 5 2 25 = 1 ⇒ y — 5 2 = 3 4 · 25 ⇒ y = ± 5 3 2 + 5

    Тогда 2 ; 5 3 2 + 5 и 2 ; — 5 3 2 + 5 являются точками касания, которые принадлежат верхнему и нижнему полуэллипсу.

    Перейдем к нахождению и разрешению уравнения эллипса относительно y . Получим, что

    x — 3 2 4 + y — 5 2 25 = 1 y — 5 2 25 = 1 — x — 3 2 4 (y — 5) 2 = 25 · 1 — x — 3 2 4 y — 5 = ± 5 · 1 — x — 3 2 4 y = 5 ± 5 2 4 — x — 3 2

    Очевидно, что верхний полуэллипс задается с помощью функции вида y = 5 + 5 2 4 — x — 3 2 , а нижний y = 5 — 5 2 4 — x — 3 2 .

    Применим стандартный алгоритм для того, чтобы составить уравнение касательной к графику функции в точке. Запишем, что уравнение для первой касательной в точке 2 ; 5 3 2 + 5 будет иметь вид

    y » = 5 + 5 2 4 — x — 3 2 » = 5 2 · 1 2 4 — (x — 3) 2 · 4 — (x — 3) 2 » = = — 5 2 · x — 3 4 — (x — 3) 2 ⇒ y » (x 0) = y » (2) = — 5 2 · 2 — 3 4 — (2 — 3) 2 = 5 2 3 ⇒ y = y » (x 0) · x — x 0 + y 0 ⇔ y = 5 2 3 (x — 2) + 5 3 2 + 5

    Получаем, что уравнение второй касательной со значением в точке
    2 ; — 5 3 2 + 5 принимает вид

    y » = 5 — 5 2 4 — (x — 3) 2 » = — 5 2 · 1 2 4 — (x — 3) 2 · 4 — (x — 3) 2 » = = 5 2 · x — 3 4 — (x — 3) 2 ⇒ y » (x 0) = y » (2) = 5 2 · 2 — 3 4 — (2 — 3) 2 = — 5 2 3 ⇒ y = y » (x 0) · x — x 0 + y 0 ⇔ y = — 5 2 3 (x — 2) — 5 3 2 + 5

    Графически касательные обозначаются так:

    Касательная к гиперболе

    Когда гипербола имеет центр в точке x c e n t e r ; y c e n t e r и вершины x c e n t e r + α ; y c e n t e r и x c e n t e r — α ; y c e n t e r , имеет место задание неравенства x — x c e n t e r 2 α 2 — y — y c e n t e r 2 b 2 = 1 , если с вершинами x c e n t e r ; y c e n t e r + b и x c e n t e r ; y c e n t e r — b , тогда задается при помощи неравенства x — x c e n t e r 2 α 2 — y — y c e n t e r 2 b 2 = — 1 .

    Гипербола может быть представлена в виде двух объединенных функций вида

    y = b a · (x — x c e n t e r) 2 — a 2 + y c e n t e r y = — b a · (x — x c e n t e r) 2 — a 2 + y c e n t e r или y = b a · (x — x c e n t e r) 2 + a 2 + y c e n t e r y = — b a · (x — x c e n t e r) 2 + a 2 + y c e n t e r

    В первом случае имеем, что касательные параллельны о у, а во втором параллельны о х.

    Отсюда следует, что для того, чтобы найти уравнение касательной к гиперболе, необходимо выяснить, какой функции принадлежит точка касания. Чтобы определить это, необходимо произвести подстановку в уравнения и проверить их на тождественность.

    Пример 7

    Составить уравнение касательной к гиперболе x — 3 2 4 — y + 3 2 9 = 1 в точке 7 ; — 3 3 — 3 .

    Решение

    Необходимо преобразовать запись решения нахождения гиперболы при помощи 2 функций. Получим, что

    x — 3 2 4 — y + 3 2 9 = 1 ⇒ y + 3 2 9 = x — 3 2 4 — 1 ⇒ y + 3 2 = 9 · x — 3 2 4 — 1 ⇒ y + 3 = 3 2 · x — 3 2 — 4 и л и y + 3 = — 3 2 · x — 3 2 — 4 ⇒ y = 3 2 · x — 3 2 — 4 — 3 y = — 3 2 · x — 3 2 — 4 — 3

    Необходимо выявить, к какой функции принадлежит заданная точка с координатами 7 ; — 3 3 — 3 .

    Очевидно, что для проверки первой функции необходимо y (7) = 3 2 · (7 — 3) 2 — 4 — 3 = 3 3 — 3 ≠ — 3 3 — 3 , тогда точка графику не принадлежит, так как равенство не выполняется.

    Для второй функции имеем, что y (7) = — 3 2 · (7 — 3) 2 — 4 — 3 = — 3 3 — 3 ≠ — 3 3 — 3 , значит, точка принадлежит заданному графику. Отсюда следует найти угловой коэффициент.

    Получаем, что

    y » = — 3 2 · (x — 3) 2 — 4 — 3 » = — 3 2 · x — 3 (x — 3) 2 — 4 ⇒ k x = y » (x 0) = — 3 2 · x 0 — 3 x 0 — 3 2 — 4 x 0 = 7 = — 3 2 · 7 — 3 7 — 3 2 — 4 = — 3

    Ответ: уравнение касательной можно представить как

    y = — 3 · x — 7 — 3 3 — 3 = — 3 · x + 4 3 — 3

    Наглядно изображается так:

    Касательная к параболе

    Чтобы составить уравнение касательной к параболе y = a x 2 + b x + c в точке x 0 , y (x 0) , необходимо использовать стандартный алгоритм, тогда уравнение примет вид y = y » (x 0) · x — x 0 + y (x 0) . Такая касательная в вершине параллельна о х.

    Следует задать параболу x = a y 2 + b y + c как объединение двух функций. Поэтому нужно разрешить уравнение относительно у. Получаем, что

    x = a y 2 + b y + c ⇔ a y 2 + b y + c — x = 0 D = b 2 — 4 a (c — x) y = — b + b 2 — 4 a (c — x) 2 a y = — b — b 2 — 4 a (c — x) 2 a

    Графически изобразим как:

    Для выяснения принадлежности точки x 0 , y (x 0) функции, нежно действовать по стандартному алгоритму. Такая касательная будет параллельна о у относительно параболы.

    Пример 8

    Написать уравнение касательной к графику x — 2 y 2 — 5 y + 3 , когда имеем угол наклона касательной 150 ° .

    Решение

    Начинаем решение с представления параболы в качестве двух функций. Получим, что

    2 y 2 — 5 y + 3 — x = 0 D = (- 5) 2 — 4 · (- 2) · (3 — x) = 49 — 8 x y = 5 + 49 — 8 x — 4 y = 5 — 49 — 8 x — 4

    Значение углового коэффициента равняется значению производной в точке x 0 этой функции и равняется тангенсу угла наклона.

    Получаем:

    k x = y » (x 0) = t g α x = t g 150 ° = — 1 3

    Отсюда определим значение х для точек касания.

    Первая функция запишется как

    y » = 5 + 49 — 8 x — 4 » = 1 49 — 8 x ⇒ y » (x 0) = 1 49 — 8 x 0 = — 1 3 ⇔ 49 — 8 x 0 = — 3

    Очевидно, что действительных корней нет, так как получили отрицательное значение. Делаем вывод, что касательной с углом 150 ° для такой функции не существует.

    Вторая функция запишется как

    y » = 5 — 49 — 8 x — 4 » = — 1 49 — 8 x ⇒ y » (x 0) = — 1 49 — 8 x 0 = — 1 3 ⇔ 49 — 8 x 0 = — 3 x 0 = 23 4 ⇒ y (x 0) = 5 — 49 — 8 · 23 4 — 4 = — 5 + 3 4

    Имеем, что точки касания — 23 4 ; — 5 + 3 4 .

    Ответ: уравнение касательной принимает вид

    y = — 1 3 · x — 23 4 + — 5 + 3 4

    Графически изобразим это таким образом:

    Если вы заметили ошибку в тексте, пожалуйста, выделите её и нажмите Ctrl+Enter

    Пусть дана функция f , которая в некоторой точке x 0 имеет конечную производную f (x 0). Тогда прямая, проходящая через точку (x 0 ; f (x 0)), имеющая угловой коэффициент f ’(x 0), называется касательной.

    А что будет, если производная в точке x 0 не существует? Возможны два варианта:

    1. Касательная к графику тоже не существует. Классический пример — функция y = |x | в точке (0; 0).
    2. Касательная становится вертикальной. Это верно, к примеру, для функции y = arcsin x в точке (1; π /2).

    Уравнение касательной

    Всякая невертикальная прямая задается уравнением вида y = kx + b , где k — угловой коэффициент. Касательная — не исключение, и чтобы составить ее уравнение в некоторой точке x 0 , достаточно знать значение функции и производной в этой точке.

    Итак, пусть дана функция y = f (x ), которая имеет производную y = f ’(x ) на отрезке . Тогда в любой точке x 0 ∈ (a ; b ) к графику этой функции можно провести касательную, которая задается уравнением:

    y = f ’(x 0) · (x − x 0) + f (x 0)

    Здесь f ’(x 0) — значение производной в точке x 0 , а f (x 0) — значение самой функции.

    Задача. Дана функция y = x 3 . Составить уравнение касательной к графику этой функции в точке x 0 = 2.

    Уравнение касательной: y = f ’(x 0) · (x − x 0) + f (x 0). Точка x 0 = 2 нам дана, а вот значения f (x 0) и f ’(x 0) придется вычислять.

    Для начала найдем значение функции. Тут все легко: f (x 0) = f (2) = 2 3 = 8;
    Теперь найдем производную: f ’(x ) = (x 3)’ = 3x 2 ;
    Подставляем в производную x 0 = 2: f ’(x 0) = f ’(2) = 3 · 2 2 = 12;
    Итого получаем: y = 12 · (x − 2) + 8 = 12x − 24 + 8 = 12x − 16.
    Это и есть уравнение касательной.

    Задача. Составить уравнение касательной к графику функции f (x ) = 2sin x + 5 в точке x 0 = π /2.

    В этот раз не будем подробно расписывать каждое действие — укажем лишь ключевые шаги. Имеем:

    f (x 0) = f (π /2) = 2sin (π /2) + 5 = 2 + 5 = 7;
    f ’(x ) = (2sin x + 5)’ = 2cos x ;
    f ’(x 0) = f ’(π /2) = 2cos (π /2) = 0;

    Уравнение касательной:

    y = 0 · (x − π /2) + 7 ⇒ y = 7

    В последнем случае прямая оказалась горизонтальной, т. к. ее угловой коэффициент k = 0. Ничего страшного в этом нет — просто мы наткнулись на точку экстремума.

    Инструкция

    Определяем угловой коэффициент касательной к кривой в точке М.
    Кривая, представляющая собой график функции y = f(x), непрерывна в некоторой окрестности точки М (включая саму точку М).

    Если значения f‘(x0) не существует, то либо касательной нет, либо она проходит вертикально. Ввиду этого, наличие производной функции в точке х0 обусловлено существованием невертикальной касательной, соприкасающейся с графиком функции в точке (х0, f(х0)). В этом случае угловой коэффициент касательной равен будет f»(х0). Таким образом, становится ясен геометрический смысл производной – расчет углового коэффициента касательной.

    Найдите значение абсциссы точки касания, которую обозначаются буквой «а». Если она совпадает с заданной точкой касательной, то «а» будет ее х-координате. Определите значение функции f(a), подставив в уравнение функции величину абсциссы.

    Определите первую производную уравнения функции f’(x) и подставьте в него значение точки «а».

    Возьмите общее уравнение касательной, которое определяется как y = f(a) = f (a)(x – a), и подставьте в него найденные значения a, f(a), f «(a). В результате будет найдено решение графика и касательной.

    Решите задачу иным способом, если заданная точка касательной не совпала с точкой касания. В этом случае необходимо в уравнение касательной вместо цифр подставить «а». После этого вместо букв «х» и «у» подставьте значение координат заданной точки. Решите получившееся уравнение, в котором «а» является неизвестной. Поставьте полученное значение в уравнение касательной.

    Составьте уравнение касательной с буквой «а», если в условии задачи задано уравнение функции и уравнение параллельной линии относительно искомой касательной. После этого необходимо производную функции , чтобы координату у точки «а». Подставьте соответствующее значение в уравнение касательной и решите функцию.

    Найти наклон прямой, касательной к кривой в заданной точке

    Если вы считаете, что контент, доступный с помощью Веб-сайта (как это определено в наших Условиях обслуживания), нарушает одно или более ваших авторских прав, пожалуйста, сообщите нам, предоставив письменное уведомление («Уведомление о нарушении»), содержащее в информацию, описанную ниже, назначенному агенту, указанному ниже. Если университетские наставники примут меры в ответ на ан Уведомление о нарушении, он предпримет добросовестную попытку связаться со стороной, предоставившей такой контент средства самого последнего адреса электронной почты, если таковой имеется, предоставленного такой стороной Varsity Tutors.

    Ваше Уведомление о нарушении может быть направлено стороне, предоставившей контент, или третьим лицам, таким как в виде ChillingEffects.org.

    Обратите внимание, что вы будете нести ответственность за ущерб (включая расходы и гонорары адвокатов), если вы существенно искажать информацию о том, что продукт или деятельность нарушают ваши авторские права. Таким образом, если вы не уверены, что содержимое находится на Веб-сайте или на который ссылается Веб-сайт, нарушает ваши авторские права, вам следует сначала обратиться к адвокату.

    Чтобы подать уведомление, выполните следующие действия:

    Вы должны включить следующее:

    Физическая или электронная подпись владельца авторских прав или лица, уполномоченного действовать от его имени; Идентификация авторских прав, которые, как утверждается, были нарушены; Описание характера и точного местонахождения контента, который, как вы утверждаете, нарушает ваши авторские права, в \ достаточно подробно, чтобы преподаватели университета могли найти и точно идентифицировать этот контент; например, мы требуем а ссылку на конкретный вопрос (а не только название вопроса), который содержит содержание и описание к какой конкретной части вопроса — изображению, ссылке, тексту и т. д. — относится ваша жалоба; Ваше имя, адрес, номер телефона и адрес электронной почты; и Заявление от вас: (а) что вы добросовестно полагаете, что использование контента, который, как вы утверждаете, нарушает ваши авторские права не разрешены законом или владельцем авторских прав или его агентом; б) что все информация, содержащаяся в вашем Уведомлении о нарушении, является точной, и (c) под страхом наказания за лжесвидетельство вы либо владельцем авторских прав, либо лицом, уполномоченным действовать от их имени.2+1 в любой точке c

    Определение наклона касательной совпадает с определением производной.

     

     

    lim           f(c + h) — f(c)

    ч->0     _______________

                            ч

     

     

    lim         2(c + h) 2 + 1 — (2c 2 + 1)

    ч —> 0    ______________________

                              ч

     

     

    lim           2(c 2  + 2ch + h 2 ) + 1 — 2c 2 — 1

    ч —> 0      __________________________

                                     ч

     

     

     

    lim           2c 2 + 4ch + 2h 2 + 1 — 2c 2 — 1

    ч —> 0     _________________________

                                      ч

     

     

    lim             4ch + 2h 2

    х—>0       __________

                           ч

     

     

    lim           2h(2c + h)

    ч —> 0      ___________                 h  отменяется.

                           ч

     

     

    lim          2(2c + h)                     подключите h=0

    ч —> 0

     

     

     

    лим         4c

    ч —> 0

     

     

     

    Наклон касательной равен 4c. Теперь точка исходной функции в любой точке c равна (c, 2c 2 + 1). Используйте эту точку, чтобы найти уравнение касательной.Нам также нужно найти b , используя форму пересечения наклона.

     

    у = мх + б

     

    2 + 1 = 4с(с) + б

     

    2 + 1 = 4с 2  + б

     

     

    -2с 2 + 1 = б

     

     

    Уравнение касательной

     

    у = 4сх + (-2с 2 + 1)

     

    у = 4сх — 2с 2  + 1

     

     

     

     

    Исчисление I. Касательные линии и скорости изменения

    Показать мобильное уведомление Показать все примечания Скрыть все примечания

    Похоже, вы используете устройство с «узкой» шириной экрана ( i. е. вы, вероятно, на мобильном телефоне). Из-за характера математики на этом сайте лучше всего просматривать в ландшафтном режиме. Если ваше устройство не находится в ландшафтном режиме, многие уравнения будут отображаться сбоку вашего устройства (должна быть возможность прокрутки, чтобы увидеть их), а некоторые пункты меню будут обрезаны из-за узкой ширины экрана.

    Раздел 2-1: Касательные линии и скорости изменения

    В этом разделе мы собираемся рассмотреть две довольно важные проблемы в изучении исчисления.Есть две причины обратить внимание на эти проблемы сейчас.

    Во-первых, обе эти проблемы приведут нас к изучению пределов, что, в конце концов, является темой этой главы. Рассмотрение этих проблем здесь позволит нам начать понимать, что такое предел и что он может рассказать нам о функции.

    Во-вторых, проблема скорости изменения, которую мы собираемся рассмотреть, является одним из наиболее важных понятий, с которыми мы столкнемся во второй главе этого курса. На самом деле, это, наверное, одна из самых важных концепций, с которыми мы столкнемся на протяжении всего курса. Итак, глядя на это сейчас, мы начинаем думать об этом с самого начала.

    Касательные линии

    Первая задача, которую мы собираемся рассмотреть, — это задача касательной. Прежде чем приступить к этой проблеме, вероятно, было бы лучше определить касательную линию.

    Касательной к функции \(f(x)\) в точке \(x = a\) называется прямая, которая касается графика функции в рассматриваемой точке и является «параллельной» (каким-то образом ) к графику в этой точке.Взгляните на график ниже.

    На этом графике линия является касательной в указанной точке, потому что она только касается графика в этой точке, а также «параллельна» графику в этой точке. Точно так же во второй показанной точке линия просто касается графика в этой точке, но она не «параллельна» графику в этой точке и, следовательно, не является касательной к графику в этой точке.

    Во второй показанной точке (точке, где линия не является касательной) мы иногда будем называть линию секущей линией .

    Мы уже несколько раз использовали слово «параллельный», и нам, вероятно, следует быть с ним немного осторожнее. В общем, мы будем думать о линии и графике как о параллельных в точке, если они оба движутся в одном направлении в этой точке. Итак, в первой точке выше график и линия движутся в одном направлении, поэтому мы будем говорить, что они параллельны в этой точке. С другой стороны, во второй точке линия и график не движутся в одном направлении, поэтому в этой точке они не параллельны.2}\) в \(х=1\). Показать решение

    Из алгебры мы знаем, что для нахождения уравнения прямой нужны либо две точки на прямой, либо одна точка на прямой и наклон прямой. Поскольку мы знаем, что находимся после касательной, у нас есть точка, которая находится на этой линии. Касательная и график функции должны соприкасаться в точке \(x\) = 1, поэтому точка \(\left( {1,f\left( 1 \right)} \right) = \left( {1,13 } \right)\) должен быть в строке.

    Вот мы и добрались до проблемы.Это все, что мы знаем о касательной. Чтобы найти касательную, нам нужна либо вторая точка, либо наклон касательной. Поскольку единственная причина, по которой нам нужна вторая точка, состоит в том, чтобы позволить нам найти наклон касательной, давайте просто сконцентрируемся на том, сможем ли мы определить наклон касательной.

    На данный момент все, что мы можем сделать, это получить оценку наклона касательной, но если мы сделаем это правильно, мы сможем получить оценку, которая на самом деле является фактической. наклон касательной.Мы сделаем это, начав с точки, которая нам нужна, назовем ее \(P = \left({1,13} \right)\). Затем мы выберем другую точку, лежащую на графике функции, назовем ее \(Q = \left( {x,f\left( x \right)} \right)\).

    Для аргументации давайте выберем \(x = 2\) и тогда вторая точка будет \(Q = \left({2,7} \right)\). Ниже приведен график функции, касательная и секущая, соединяющая \(P\) и \(Q\).

    Из этого графика видно, что секущая и касательная в чем-то похожи, поэтому наклон секущей должен быть несколько близок к фактическому наклону касательной.Итак, в качестве оценки наклона касательной мы можем использовать наклон секущей, назовем его \({m_{PQ}}\), то есть

    \[{m_{PQ}} = \frac{{f\left( 2 \right) — f\left( 1 \right)}}{{2 — 1}} = \frac{{7 — 13}}{ 1} = — 6\]

    Теперь, если бы нас не слишком интересовала точность, мы могли бы сказать, что этого достаточно, и использовать это как оценку наклона касательной. Однако хотелось бы, чтобы оценка была хоть как-то близка к фактическому значению.Таким образом, чтобы получить лучшую оценку, мы можем взять \(x\), который ближе к \(x = 1\), и повторить вышеописанную работу, чтобы получить новую оценку наклона. Затем мы могли бы взять третье значение \(x\) еще ближе и получить еще лучшую оценку.

    Другими словами, по мере того, как мы приближаем \(Q\) к \(P\), наклон секущей, соединяющей \(Q\) и \(P\), должен становиться все ближе и ближе к наклону касательная линия. Если вы просматриваете это в Интернете, на изображении ниже показан этот процесс.

    Как вы можете видеть (если вы читаете это в Интернете), по мере того, как мы перемещали \(Q\) все ближе и ближе к \(P\), секущие линии начинают все больше и больше походить на касательную и так далее. приблизительные наклоны ( т.е. наклоны секущих) все ближе и ближе к точному наклону. Кроме того, не беспокойтесь о том, как я получил точные или приблизительные наклоны. Вскоре мы вычислим приблизительные наклоны и сможем вычислить точный наклон на нескольких участках.

    На этом рисунке мы рассмотрели только \(Q\), которые были справа от \(P\), но мы могли бы так же легко использовать \(Q\), которые были слева от \( P\), и мы получили бы такие же результаты. На самом деле, мы всегда должны смотреть на \(Q\), которые находятся по обе стороны от \(P\). В этом случае одно и то же происходит по обе стороны от \(P\). Однако в конечном итоге мы увидим, что этого не должно произойти. 2}}}{{х — 1}}\]

    Теперь давайте выберем некоторые значения \(x\), все ближе и ближе к \(x = 1\), подключим и получим наклоны.

    \(х\) \({м_{PQ}}\) \(х\) \({м_{PQ}}\)
    2 -6 0 -2
    1,5 -5 0,5 -3
    1.1 -4,2 0,9 -3,8
    1,01 -4,02 0,99 -3,98
    1,001 -4.002 0,999 -3,998
    1. 0001 -4.0002 0,9999 -3,9998

    Итак, если мы возьмем \(x\) справа от 1 и переместим их очень близко к 1, окажется, что наклон секущих приближается к -4. Точно так же, если мы возьмем \(x\) слева от 1 и переместим их очень близко к 1, наклон секущих снова окажется приближающимся к -4.

    Основываясь на этом свидетельстве, кажется, что наклоны секущих приближаются к -4, когда мы приближаемся к \(x = 1\), поэтому мы оценим, что наклон касательной также равен -4.2}\) в \(x=1\) равно

    \[y = 13 — 4\влево( {x — 1} \вправо) = — 4x + 17\]

    Есть несколько важных замечаний по поводу нашей работы выше. Во-первых, мы рассмотрели точки, находящиеся по обе стороны от \(x = 1\). В такого рода процессах важно никогда не предполагать, что то, что происходит на одной стороне точки, будет происходить и на другой стороне. Мы всегда должны смотреть на то, что происходит по обе стороны от точки. В этом примере мы могли бы набросать график и, исходя из этого, предположить, что то, что происходит с одной стороны, будет происходить и с другой, но обычно у нас не будет графиков перед собой или мы не сможем легко их получить.

    Далее, обратите внимание, что когда мы говорим, что собираемся приблизиться к рассматриваемой точке, мы имеем в виду, что собираемся приблизиться очень близко, и мы также использовали больше, чем пару точек. Мы никогда не должны пытаться определить тренд, основываясь на паре точек, которые на самом деле не так уж близки к рассматриваемой точке.

    Следующее, на что следует обратить внимание, — это скорее предупреждение, чем что-либо еще. Значения \({m_{PQ}}\) в этом примере были довольно «хорошими», и было довольно ясно, к какому значению они приближаются после пары вычислений.В большинстве случаев этого не будет. Большинство значений будут гораздо «беспорядочнее», и вам часто потребуется довольно много вычислений, чтобы получить оценку. Вы всегда должны использовать не менее четырех точек с каждой стороны, чтобы получить оценку. Двух баллов никогда не бывает достаточно для получения хорошей оценки, и трех баллов также часто недостаточно для получения хорошей оценки. Как правило, вы выбираете точки все ближе и ближе к точке, на которую смотрите, пока изменение значения между двумя последовательными точками не станет очень маленьким.

    Наконец, мы искали то, что происходило в точке \(x = 1\), и мы не могли фактически включить \(x = 1\) в нашу формулу для наклона. Несмотря на это ограничение, мы смогли получить некоторую информацию о том, что происходит в точке \(x = 1\), просто взглянув на то, что происходит вокруг точки \(x = 1\). Это важнее, чем вы можете себе представить на первый взгляд, и мы подробно обсудим этот момент в следующих разделах.

    Прежде чем двигаться дальше, давайте кратко рассмотрим, что мы сделали в приведенном выше примере.Мы хотели, чтобы касательная шла к \(f\left( x \right)\) в точке \(x = a\). Во-первых, мы знаем, что точка \(P = \left( {a,f\left( a \right)} \right)\) будет на касательной. Затем мы возьмем вторую точку на графике функции, назовем ее \(Q = \left( {x,f\left( x \right)} \right)\) и вычислим наклон кривой линия, соединяющая \(P\) и \(Q\) следующим образом,

    \[{m_{PQ}} = \frac{{f\left( x \right) — f\left( a \right)}}{{x — a}}\]

    Затем мы берем значения \(x\), которые все ближе и ближе к \(x = a\) (убедившись, что мы смотрим на \(x\) по обе стороны от \(x = a\) и используем этот список значений для оценки наклона касательной, \(m\).

    Тогда касательная будет равна

    . \[y = f\left( a \right) + m\left( {x — a} \right)\]
    Скорость изменения

    Следующая проблема, которую нам нужно рассмотреть, — это проблема скорости изменения. Как упоминалось ранее, это окажется одной из самых важных концепций, которые мы будем рассматривать на протяжении всего курса.

    Здесь мы собираемся рассмотреть функцию \(f\left( x \right)\), представляющую некоторую величину, которая изменяется при изменении \(x\). Например, возможно, \(f\left( x \right)\) представляет собой количество воды в накопительном баке через \(x\) минут. Или, может быть, \(f\left( x \right)\) — это расстояние, пройденное автомобилем за \(x\) часов. В обоих этих примерах мы использовали \(x\) для представления времени. Конечно, \(x\) не обязательно должно представлять время, но это дает примеры, которые легко визуализировать.

    Здесь мы хотим определить, насколько быстро \(f\left( x \right)\) меняется в какой-то момент, скажем, \(x = a\). Это называется мгновенной скоростью изменения или иногда просто скоростью изменения \(f\left( x \right)\) в \(x = a\).

    Как и в случае с задачей касательной линии, все, что мы сейчас можем сделать, — это оценить скорость изменения. Итак, давайте продолжим с приведенными выше примерами и будем думать о \(f\left( x \right)\) как о чем-то, что изменяется во времени, а \(x\) является измерением времени. Опять же, \(x\) не обязательно представляет время, но это немного облегчит объяснение. Хотя мы не можем вычислить мгновенную скорость изменения, мы можем найти среднюю скорость изменения.

    Чтобы вычислить среднюю скорость изменения \(f\left( x \right)\) в \(x = a\), все, что нам нужно сделать, это выбрать другую точку, скажем, \(x\), а затем средняя скорость изменения составит

    \[\begin{align*}А.Р.К. & = \frac{{{\mbox{изменение}}f\left( x \right)}}{{{\mbox{изменение}}x}}\\ & = \frac{{f\left( x \right) — f\left( a \right)}}{{x — a}}\end{align*}\]

    Тогда для оценки мгновенной скорости изменения при \(x = a\) все, что нам нужно сделать, это выбрать значения \(x\), все ближе и ближе к \(x = a\) (не забудьте выбрать их по обе стороны от \(x = a\)) и вычислить значения \(A.2} + 25}}{{т — 5}}\]

    Чтобы оценить мгновенную скорость изменения объема при \(t = 5\), нам просто нужно выбрать значения \(t\), которые все ближе и ближе к \(t = 5\). Вот таблица значений \(t\) и средней скорости изменения этих значений.

    \(т\) \(АРК\) \(т\) \(АРК\)
    6 25.0 4 7,0
    5,5 19,75 4,5 10,75
    5.1 15,91 4,9 14.11
    5,01 15.0901 4,99 14.9101
    5,001 15.009001 4,999 14.9

    5.0001 15.000 4,9999 14. 99

    1

    Итак, из этой таблицы видно, что средняя скорость изменения приближается к 15, и поэтому мы можем оценить, что мгновенная скорость изменения на данный момент равна 15.

    Итак, что же это говорит нам об объеме в точке \(t = 5\)? Положим несколько единиц на ответ сверху. Это может помочь нам увидеть, что происходит с томом в этот момент. Предположим, что единицы объема были в см 3 . Тогда единицы скорости изменения (как средней, так и мгновенной) равны см 3 /час.

    Мы подсчитали, что при \(t = 5\) объем изменяется со скоростью 15 см 3 /час.Это означает, что при \(t = 5\) объем изменяется так, что если бы скорость была постоянной, то через час в баллоне было бы на 15 см 3 воздуха больше, чем было при \( т = 5\).

    Однако здесь нужно быть осторожным. На самом деле через час воздуха в шаре, вероятно, не останется еще 15 см 3 . Скорость, с которой изменяется объем, как правило, непостоянна, поэтому мы не можем реально определить, каким будет объем через следующий час.Что мы можем сказать, так это то, что объем увеличивается, поскольку мгновенная скорость изменения положительна, и если бы у нас были скорости изменения для других значений \(t\), мы могли бы сравнить числа и посмотреть, быстрее ли скорость изменения или медленнее в других точках.

    Например, при \(t = 4\) мгновенная скорость изменения равна 0 см 3 /час, а при \(t = 3\) мгновенная скорость изменения равна -9 см 3 /час. Мы предоставим вам проверить эти скорости изменения.На самом деле, это было бы хорошим упражнением, чтобы увидеть, сможете ли вы построить таблицу значений, которая подтвердит наши утверждения об этих темпах изменений.

    В любом случае, вернемся к примеру. При \(t = 4\) скорость изменения равна нулю, поэтому в этот момент времени объем вообще не меняется. Это не значит, что он не изменится в будущем. Это просто означает, что ровно при \(t = 4\) громкость не меняется. Точно так же в точке \(t = 3\) объем уменьшается, поскольку скорость изменения в этой точке отрицательна.Мы также можем сказать, что, независимо от аспектов увеличения/уменьшения скорости изменения, объем воздушного шара изменяется быстрее при \(t = 5\), чем при \(t = 3\), поскольку 15 больше чем 9.

    Мы еще поговорим о скорости изменений, когда перейдем к следующей главе.

    Проблема скорости

    Давайте кратко рассмотрим проблему скорости. Многие книги по математическому анализу рассматривают это как отдельную проблему. Однако нам нравится думать об этом как о частном случае проблемы скорости изменения.В задаче о скорости нам дана функция положения объекта \(f\left( t \right)\), которая дает положение объекта в момент времени \(t\). Затем, чтобы вычислить мгновенную скорость объекта, нам просто нужно вспомнить, что скорость — это не что иное, как скорость изменения положения.

    Другими словами, чтобы оценить мгновенную скорость, мы должны сначала вычислить среднюю скорость,

    \[\begin{align*}А.В. & = \frac{{{\mbox{изменение положения}}}}{{{\mbox{пройденное время}}}}\\ & \\ & = \frac{{f\left( t \right) — f \left( a \right)}}{{t — a}}\end{align*}\]

    , а затем подбирайте значения \(t\) все ближе и ближе к \(t = a\) и используйте эти значения для оценки мгновенной скорости.

    Изменение обозначения

    Прежде чем двигаться дальше, нам нужно сделать еще одну вещь в этом разделе. Основная цель этого раздела состояла в том, чтобы познакомить нас с парой ключевых концепций и идей, которые мы увидим в первой части этого курса, а также помочь нам начать путь к ограничениям.

    Прежде чем мы официально перейдем к ограничениям, давайте вернемся назад и проделаем небольшую работу, которая свяжет обе (или все три, если включить скорость как отдельную проблему) проблемы в более общую концепцию.

    Во-первых, обратите внимание, что независимо от того, нужна ли нам касательная, мгновенная скорость изменения или мгновенная скорость, каждый из них сводился к использованию одной и той же формулы. А именно

    \begin{equation}\frac{{f\left( x \right) — f\left( a \right)}}{{x — a}} \label{eq:eq1}\end{equation}

    Это должно означать, что все три проблемы на самом деле являются одной и той же проблемой. На самом деле это так, как мы увидим в следующей главе. Мы действительно работаем над одной и той же проблемой в каждом из этих случаев, единственная разница заключается в интерпретации результатов.

    При подготовке к следующему разделу, где мы обсудим это более подробно, нам нужно быстро изменить обозначения. Это проще сделать здесь, так как мы уже потратили изрядное количество времени на эти проблемы.

    Во всех этих задачах мы хотели определить, что происходит в точке \(x = a\). Для этого мы выбрали другое значение \(x\) и подключили его к \(\eqref{eq:eq1}\). Для того, что мы делали здесь, это, вероятно, самый интуитивный способ сделать это.Однако, когда мы начнем рассматривать эти проблемы как одну проблему, \(\eqref{eq:eq1}\) не будет лучшей формулой для работы.

    Вместо этого мы сначала определим, насколько далеко от \(x = a\) мы хотим двигаться, а затем определим нашу новую точку на основе этого решения. Итак, если мы хотим переместиться на расстояние \(h\) от \(x = a\), новая точка будет \(x = a + h\). Это показано на эскизе ниже.

    Как мы видели в нашей работе выше, важно брать значения \(x\), которые являются обеими сторонами \(x = a\).Этот способ выбора нового значения \(x\) сделает это за нас, как мы можем видеть на скетче выше. Если \(h > 0\), мы получим значение \(x\), которое находится справа от \(x = a\), а если \(h < 0\), мы получим значения \(x\) которые находятся слева от \(x = a\), и оба задаются как \(x = a + h\).

    Теперь, с этим новым способом получения второго значения \(x\) \(\eqref{eq:eq1}\) станет

    \[\frac{{f\left( x \right) — f\left( a \right)}}{{x — a}} = \frac{{f\left( {a + h} \right) — f\left( a \right)}}{{a + h — a}} = \frac{{f\left( {a + h} \right) — f\left( a \right)}}{h} \]

    Теперь это для определенного значения \(x\), т. е.е. \(x = a\), и мы редко будем рассматривать их при конкретных значениях \(x\). Итак, мы делаем последний шаг в приведенном выше уравнении и заменяем \(a\) на \(x\), чтобы получить

    \[\frac{{f\left( {x + h} \right) — f\left( x \right)}}{h}\]

    Это дает нам формулу для общего значения \(x\), и на первый взгляд может показаться, что это будет слишком сложный способ работы с этим материалом. Однако, как мы увидим, часто будет проще иметь дело с этой формой, чем с исходной формой, \(\eqref{eq:eq1}\).2}$ (верхняя полуокружность радиуса 25 с центром в начале координат). При $x=7$ получаем, что $\ds y=\sqrt{625-49}=24$. Предположим, мы хотим узнать, насколько изменится $y$ при небольшом увеличении $x$, скажем 7.1 или 7.01.

    В случае прямой $y=mx+b$ наклон $m=\Delta y/\Delta x$ измеряет изменение $y$ на единицу изменения $x$. Это может быть интерпретируется как мера «чувствительности»; например, если $y=100x+5$, небольшое изменение $x$ соответствует изменению на сто раз больше $y$, поэтому $y$ очень чувствительна к изменениям $x$. 2}\ок 23,9706-24=-0,0294.\кр }$$ Таким образом, $\Delta y/\Delta x\приблизительно -0,0294/0,1=-0,294$. Это означает, что $y$ изменяется менее чем на одну треть от изменения $x$, так что, очевидно, $y$ не очень чувствителен к изменению $x$ при $x=7$. Мы говорим «видимо» здесь, потому что мы действительно не знаем, что происходит между 7 и $7,1$. Возможно, $y$ резко меняется, когда $x$ проходит через значения с 7 до 7,1$, но при 7,1$ $y$ как раз близок к своему стоимость в $7$. На самом деле это не так для этого конкретного функционируют, но мы еще не знаем, почему.$\квадрат$

    Один из способов интерпретировать приведенный выше расчет — это ссылка на линию. Мы вычислили наклон прямой через $(7,24)$ и $(7.1,23.9706)$, называемая хордой окружности. В общем случае, если провести хорду из точки $(7,24)$ в ближайшую точка на полуокружности $(7+\Delta x,\,f(7+\Delta x))$, наклон этой хорда — это так называемое разностное частное $$ \hbox{наклон хорды}={f(7+\Delta x)-f(7)\over \Delta x}= {\ sqrt {625-(7+\Delta x)^2}-24\over \Delta x}. $$ Например, если $x$ изменится только с 7 на 7,01, то коэффициент разности (наклон хорды) примерно равен $(23,997081-24)/0,01=-0,2919$. Это немного менее круто, чем хорда от $(7,24)$ до $(7,1,23,9706)$.

    Когда второе значение $7+\Delta x$ приближается к 7, соединение хорды $(7,f(7))$ до $(7+\Delta x,f(7+\Delta x))$ немного сдвигается. В виде показано на рис. 2.1.1, по мере того, как $\Delta x$ становится меньше и меньше, хорда, соединяющая $(7,24)$ с $(7+\Delta x,f(7+\Delta x))$ становится все ближе и ближе к касательная к окружности в точке $(7,24)$.(Напомним, что касательная — это линия, которая касается окружности в этой точке, т. е. не пересекается с окружностью ни в одной второй точке.) Таким образом, поскольку $\Delta x$ становится все меньше и меньше, наклон $\Delta y/\Delta x$ хорда все ближе и ближе к наклону касательной. Вы можете перетащить красную точку с надписью $f(x+\Delta x)$, чтобы посмотреть, как это происходит; фиксированная красная линия является касательной. 2}$ (зеленая).Вы можете перетащить любую красную точку в изменить точку касания или хорду.

    Пока мы нашли наклоны двух хорд, которые должны быть близки к наклон касательной, но каков наклон касательной точно линия? Так как касательная касается окружности только в одном точки, мы никогда не сможем вычислить ее наклон напрямую, используя две «известные» точки на линии. Нам нужен способ захвата что происходит с наклонами хорд, когда они становятся «ближе и ближе к линии касания.2}+24=48$. Следовательно, дробь очень близка к $-14/48 = -7/24\конг -0,29167$? Это, безусловно, кажется разумным, и в на самом деле это правда: по мере того, как $\Delta x$ становится все ближе и ближе к нулю, Коэффициент разности на самом деле становится все ближе и ближе к $-7/24$, и так что наклон касательной точно равен $-7/24$.

    А как насчет наклона касательной при $x=12$? ну 12 не может быть все, что отличается от 7; нам просто нужно повторить расчет с 12 вместо 7. Это будет несложно, но немного скучный. 2}$. В какой-то момент, скажем, $x=7$, мы говорим, что $f'(7)=-7/24$ или «$f$ простое число 7 равно $-7/24$» или «производная от $f$ в 7 составляет $-7/24$.»

    Подводя итог, мы вычисляем производную от $f(x)$, формируя коэффициент разности $$\eqalignno{ &{f(x+\Delta x)-f(x)\over \Delta x},& (2.1.1)\кр }$$ который является наклоном линии, то мы выясняем, что происходит, когда $\Delta x$ становится очень близким к 0.

    Мы должны отметить, что в частный случай круга, есть простой способ найти производная.2}}$, как и раньше. НЕ всегда верно, что касательная перпендикулярна линии из начала координат — не используйте этот ярлык в любых других обстоятельствах.

    Как и выше, и, как вы могли ожидать, для различных значений $x$ мы обычно получают разные значения производной $f'(x)$. Может быть что производная всегда имеет одно и то же значение? Это будет означать, что наклон $f$, или наклон его касательной, одинаков повсюду. Одна кривая, которая всегда имеет один и тот же наклон, является линией; Это кажется странным говорить о касательной линии к прямой, но если это делает смысл вообще касательной линии должен быть самой линией. 2}$ в тексте для каждого из следующих $x$: (a) 20, (б) 24, (в) $-7$, (г) $-15$.Нарисуйте график верхней полуокружности, и проведите касательную в каждой из этих четырех точек. (отвечать)

    Пример 2.1.3 Нарисуйте график функции $y=f(x)=1/x$ между $x=1/2$ и $x=4$. Найдите наклон хорды между (a) $x=3$ и $x=3,1$, (b) $x=3$ и $x=3,01$, (c) $x=3$ и $x=3,001$. Теперь используйте алгебру, чтобы найти простой формула наклона хорды между $(3,f(3))$ и $(3+\Delta х,f(3+\Дельта х))$. Определите, что происходит, когда $\Delta x$ приближается к 0. На графике $y=1/x$ проведите прямую через точку $(3,1/3)$, наклон которой представляет собой это предельное значение разностного отношения как $\Delta x$ приближается к 0.3$ провести прямую через точка $(1,1)$, наклон которой равен только что найденному значению. (отвечать)

    Пример 2.1.6 Найдите алгебраическое выражение для разностного отношения $(f(x+\Delta x)-f(x))/\Delta x$, когда $f(x)=mx+b$. Упростите выражение как насколько это возможно. Затем определите, что происходит, когда $\Delta x$ приближается к 0. Это значение равно $f'(x)$. (отвечать)

    Пример 2.1.7 Нарисуйте единичный круг. Обсудите поведение склона касательной под разными углами по окружности.2$. При каких значениях $x$ на положителен ли наклон касательной параболы? Отрицательный? Что вы заметили на графике в точках, где знак наклон изменяется с положительного на отрицательный и наоборот?

    1.8: Аппроксимация касательной

    Среди всех функций линейные функции самые простые. Одно из важных следствий дифференцируемости функции \(y = f (x)\) в точке \((a, f (a))\) состоит в том, что вблизи функция \(y = f ( x )\) локально линейна и имеет вид своей касательной в этой точке.В определенных обстоятельствах это позволяет нам аппроксимировать исходную функцию \(f\) более простой функцией \(L\), которая является линейной: это может быть выгодно, когда у нас есть ограниченная информация о \(f\) или когда \(f \) является вычислительно или алгебраически сложным. {\prime} (a) (x — a) + f (a)\).{ \prime } ( x )\): мы должны тщательно различать эти выражения. Каждый раз, когда мы находим касательную, нам нужно вычислять функцию и ее производную при фиксированном \(a\)-значении.

    На рис. 1.8.2 мы видим размеченный участок графика функции \(f\) и ее касательную в точке \((a, f (a))\). Обратите внимание, что при увеличении масштаба мы видим более четко выделенную локальную линейность \(f\), поскольку функция и ее касательная почти неразличимы вблизи. Это также можно увидеть динамически в апплете Java по адресу http://gvsu.образование / с / 6J.

    Рисунок \(\PageIndex{2}\) : Функция \(y = f (x)\) и ее касательная в точке \((a, f (a))\ ) : слева издалека и справа вблизи. Справа мы обозначаем функцию касательной прямой как \(y = L(x)\) и замечаем, что для \(x\) около \((a, f (a))≈ L(x) \) .

    Локальная линеаризация

    Небольшое изменение перспективы и обозначений позволит нам более точно обсудить, как касательная к \(y = f (x)\) в точке \((a, f (a))\) аппроксимирует \(f\ ) вблизи \(х = а\). { \prime } ( а ) ( х — а ) + ж ( а )\]

    локальная линеаризация \(f\) в точке\((a, f (a))\). В этих обозначениях особенно важно отметить, что \(L(x)\) есть не что иное, как новое имя для касательной, и что для \(x\), близких к \(a\), мы имеем, что \( f(x) ≈ L(x)\).

    Скажем, например, что мы знаем, что функция \(y = f (x)\) имеет аппроксимацию касательной, заданную выражением \(L(x) = 3 − 2(x − 1)\) в точке ( 1, 3), но больше ничего о функции \(f\) мы не знаем.{ \prime } ( a ) ( x — a ) \text { для } x \text { рядом } a\]

    Следующее задание исследует некоторые дополнительные важные свойства локальной линеаризации \(y = L(x)\) функции \(f\) при заданном \(a\)-значении.

    Активность \(\PageIndex{2}\)

    Предположим, что известно, что для данной дифференцируемой функции \(y = g(x)\) ее локальная линеаризация в точке, где \(a\) = −1, определяется выражением \(L ( x ) = — 2 + 3 (х + 1)\).

    1. Вычислите значения \(L\)(−1) и \(L ^ { \prime } ( — 1 )\). { \prime \prime } ( — 1 ) = 2\). Что это говорит вам о графике \(y = g(x)\) в точке \(a = −1\)?
    2. Для \(x\) вблизи −1 нарисуйте график локальной линеаризации \(y = L(x)\), а также возможный график \(y = g(x)\) на осях, указанных в Рисунок 1.8.3.

    Рисунок \(\PageIndex{3}\) : Оси построения \( y = L(x)\) и \(y = g(x)\) .

    Как мы видели в примере, представленном в Упражнении 1.{ \prime } ( а )\). Таким образом, мы видим, что \(L\) является линейной функцией, которая имеет то же значение и тот же наклон, что и функция \(f\) в точке \((a, f (a))\).

    В ситуациях, когда мы знаем линейную аппроксимацию \(y = L(x)\), мы, следовательно, знаем исходное значение функции и наклон в точке касания. Однако остается неизвестным форма функции f в точке касания. По сути, есть четыре возможности, перечисленные на рис. 1.8.4.

    Рисунок \(\PageIndex{4}\): Четыре возможных графика для нелинейной дифференцируемой функции и ее расположение относительно касательной в точке. {\prime} ( x )\) имеет график, показанный на рисунке 1.{ \prime \prime } ( x )\) в правой сетке на рис. 1.8.5; обозначьте его соответствующим образом.

    (d) Является ли наклон касательной к \(y = f(x)\) возрастающим, убывающим или ни тем, ни другим, когда \(x = 2\)? Объяснять.

    (e) Нарисуйте возможный график \(y = f(x)\) вблизи \(x = 2\2 на левой сетке на рис. 1.8.5. Включите эскиз \(y = L(x)\ ) (найдено в части (a)). Объясните, откуда вы знаете, что график \(y = f(x)\) выглядит так, как вы его нарисовали.

    (f) Ваша оценка в (b) завышает или занижает истинное значение \(f(2.07)\)? Почему?

    Идея о том, что дифференцируемая функция выглядит линейной и может быть хорошо аппроксимирована линейной функцией, является важной и находит широкое применение в исчислении. Например, аппроксимируя функцию ее локальной линеаризацией, можно разработать эффективный алгоритм оценки нулей функции. Локальная линейность также помогает нам лучше понять некоторые сложные ограничения. Например, мы видели, что такой предел, как

    \[\lim _ { x \rightarrow 0 } \ frac { \ sin ( x ) } { x } \]

    Число

    \(x\) неопределенно, потому что его числитель и знаменатель стремятся к \(0\).Хотя нет никакой алгебры, которую мы могли бы сделать, чтобы упростить \( \frac { \sin ( x ) } { x }\), просто показать, что линеаризация \(f (x) = sin(x)\) в точке \((0, 0)\) определяется выражением \(L(x) = x\). Следовательно, для значений \(x\) вблизи \(0\) \(sin(x) ≈ x\). Таким образом, для значений \(x\) вблизи \(0\)

    \[\frac {\sin ( x ) } { x } \ приблизительно \ frac { x } { x } = 1,\]

    , что делает правдоподобным тот факт, что

    \[\lim _ { x \rightarrow 0 } \ frac { \ sin ( x ) } { x } = 1 \]

    Эти идеи и другие приложения локальной линейности будут исследованы позже в нашей работе.

    2. Наклон касательной к кривой (численный подход)

    М. Борна

    Поскольку мы можем моделировать многие физические проблемы, используя кривые, важно получить представление о наклонах кривых в различных точках и о том, что означает наклон в реальных приложениях.

    ПРИМЕЧАНИЕ

    В этом разделе мы покажем вам один из исторических подходов к нахождению наклонов касательных до того, как была разработана дифференциация.Это для того, чтобы дать вам представление о том, как это работает.

    Если вы хотите узнать, как находить наклоны (градиенты) касательных непосредственно с помощью производных, перейдите к разделу Касательные и нормали в главе «Приложения дифференцирования».

    Помните: Пытаемся найти скорость изменения одной переменной по сравнению с другой.

    Приложения включают:

    • Изменение температуры в определенное время
    • Скорость падающего объекта в определенное время
    • Ток в цепи в определенное время
    • Изменение цен на фондовом рынке в определенное время
    • Рост населения в определенное время
    • Повышение температуры по мере увеличения плотности газа

    Позже мы увидим, как найти эти скорости изменения, дифференцируя функцию и подставляя значение. Сейчас мы собираемся найти скорость изменения численно (то есть, подставляя числа, пока не найдем приемлемое приближение).

    Мы рассматриваем общий случай и пишем наши функции с участием знакомые x (независимые) и y (зависимые) переменные.

    Наклон кривой y = f ( x ) в точке P означает наклон касательной в точке P .Нам надо найти этот наклон для решения многих задач, поскольку он сообщает нам скорость изменения в конкретный момент времени. мгновенное.

    [Мы пишем y = f ( x ) на кривой, поскольку y является функцией x . То есть, поскольку x варьируется, y также меняется.]

    Дельта-обозначение

    В этой работе мы пишем

    • изменить в г как Δ у
    • изменение x как Δ x

    По определению уклон определяется как:

    `m=(текст(изменение в)\ y)/(текст(изменение в)\ x)=(Deltay)/(Deltax)=(y_2-y_1)/(x_2-x_1)`

    Мы используем это, чтобы найти численное решение наклона кривой.

    Пример

    Найдите наклон кривой y = x 2 в точке `(2,4)`, используя числовой метод .

    Решение

    Начнем с точки `Q(1, 1)`, которая находится где-то рядом с `P(2,4)`:

    Наклон PQ определяется как:

    `m=(y_2-y_1)/(x_2-x_1)`

    `=(4-1)/(2-1)`

    `=3`

    Теперь мы перемещаем Q дальше по кривой так, чтобы она оказалась ближе к P .Давайте использовать `Q(1.5,2.25)`, который ближе к `П(2,4)`:

    Наклон PQ теперь определяется как:

    `m=(y_2-y_1)/(x_2-x_1)`

    `=(4-2,25)/(2-1,5)`

    `=3,5`

    Мы видим, что это уже довольно хорошее приближение к касательной в точке P , но недостаточно хорошее.

    Теперь мы переместим Q еще ближе к P, скажем `Q(1.9,3.61)`.

    Теперь у нас есть:

    Итак,

    `m=(y_2-y_1)/(x_2-x_1)`

    `=(4-3. 61)/(2-1,9)`

    `=3,9`

    Мы видим, что мы очень близки к требуемому наклону.

    Теперь, если Q перемещается в `(1.99,3.9601)`, тогда наклон PQ равен `3.99`.

    Если Q равно `(1,999,3,996001)`, то наклон равен `3,999`.

    Очевидно, что при `x → 2` наклон `PQ → 4`. Но заметьте, что на самом деле мы не можем принять `x = 2`, так как дробь для m будет иметь 0 внизу, и поэтому она будет неопределенной.

    Мы обнаружили, что скорость изменения y по отношению к x составляет «4» единицы в точке «x = 2».

    Исследовать

    Изучите этот пример с помощью интерактивного апплета на следующей странице:

    3. Производная от первых принципов.

    Теперь мы расширим этот численный подход так что мы можем найти наклон любой непрерывной кривой, если мы знаем функцию. Мы узнаем об алгебраическом подходе, который можно использовать для большинства функций.

    Как найти уравнения касательной и нормали

    Краткий обзор

    • Чтобы найти уравнение прямой, вам нужны точка и наклон.
    • Наклон касательной представляет собой значение производной в точке касания.
    • Нормальная линия — это линия, перпендикулярная касательной и проходящая через точку касания.2) = 12 $$

      Наклон касательной равен $$m = 12$$.

      Шаг 3

      Найдите точечно-наклонную форму прямой с наклоном $$m = 12$$, проходящей через точку $$(2,8)$$.

      $$ \начать{выравнивать*} y — y_1 & = m(x-x_1)\\[6pt] у — 8 и = 12 (х-2) \конец{выравнивание*} $$

      Отвечать

      $$y — 8 = 12(x-2)$$

      Для справки, вот график функции и касательной, которую мы только что нашли. 2 — х$$. Найдите уравнение касательной с наклоном $$m = -3$$.

      Шаг 1

      Найдите производную.

      $$ f'(x) = 2x -1 $$

      Шаг 2

      Найдите значение $$x$$, где $$f'(x)$$ равно наклону.

      $$ \начать{выравнивать*} f'(x) & = 2x -1\\[6pt] -3 & = 2x -1\\[6pt] -2 & = 2x\\[6pt] х & = -1 \конец{выравнивание*} $$

      Шаг 3

      Найдите точку на функции, где $$x = -1$$.2 — (-1) = 1 + 1 = 2 $$

      Точка $$(-1, 2)$$.

      Шаг 4

      Найдите уравнение прямой, проходящей через точку $$(-1,2)$$ с наклоном $$m=-3$$.

      $$ \начать{выравнивать*} y -y_1 & = m(x-x_1)\\[6pt] у — 2 & = -3(х — (-1))\\[6pt] у — 2 и = -3 (х + 1) \конец{выравнивание*} $$

      Отвечать

      $$ у — 2 = -3(х+1) $$

      Для справки, вот график функции и касательной, которую мы только что нашли.2 & = 12\\[6pt] y & = \pm\sqrt{12}\\[6pt] y & = \pm\sqrt{4\cdot 3}\\[6pt] у & = \pm2\sqrt 3 \конец{выравнивание*} $$

      Поскольку в задаче указано, что нас интересует $$y>0$$, мы используем $$y = 2\sqrt 3$$.

      Точка касания $$(2, 2\sqrt 3)$$.

      Шаг 2

      Найдите уравнение для $$\frac{dy}{dx}$$.

      Поскольку уравнение определено неявно, мы используем неявное дифференцирование.

      $$ \начать{выравнивать*} 2x + 2y\,\frac{dy}{dx} & = 0\\[6pt] 2y\,\frac{dy}{dx} & = -2x\\[6pt] \frac{dy}{dx} & = -\frac{2x}{2y}\\[6pt] \frac{dy}{dx} & = -\frac{dy} x y \конец{выравнивание*} $$

      Шаг 3

      Найдите наклон касательной в точке касания.

      В точке $$(2,2\sqrt 3)$$ наклон касательной равен

      $$ \начать{выравнивать*} \ frac {dy} {dx} \ bigg | _ {(\ blue {2}, \ red {2 \ sqrt 3})} & = — \ frac {\ blue 2} {\ red {2 \ sqrt 3}} \\[6pt] & = — \ frac 1 {\ sqrt 3} \\ [6pt] & = — \ frac 1 {\ sqrt 3} \ cdot \ blue {\ frac {\ sqrt 3} {\ sqrt 3}} \\ [6pt] & = -\frac{\sqrt 3} 3 \конец{выравнивание*} $$

      Наклон касательной равен $$m = -\frac{\sqrt 3} 3$$.

      Шаг 4

      Найдите уравнение касательной через $$(2,2\sqrt 3)$$ с наклоном $$m=-\frac{\sqrt 3} 3$$.

      В точке $$(2,2\sqrt 3)$$ наклон касательной равен

      $$ \начать{выравнивать*} y — y_1 & = m(x-x_1)\\[6pt] y — 2 \ sqrt 3 & = — \ frac {\ sqrt 3} 3 (x-2) \конец{выравнивание*} $$

      Отвечать

      Уравнение касательной: $$y — 2\sqrt 3 = -\frac{\sqrt 3} 3(x-2)$$

      Для справки график кривой и найденной нами касательной линии показан ниже.

      Нормальные линии

      Предположим, у нас есть касательная к функции. Функция и касательная пересекаются в точке касания. Линия, проходящая через ту же точку, которая перпендикулярна касательной, называется нормальной линией .

      Напомним, что когда две прямые перпендикулярны, их наклоны являются отрицательными обратными величинами.Поскольку наклон касательной равен $$m = f'(x)$$, наклон нормальной линии равен $$m = -\frac 1 {f'(x)}$$.

      Пример 4

      Предположим, что $$f(x) = \cos x$$. Найдите уравнение прямой, нормальной к функции в точке $$x = \frac \pi 6$$.

      Шаг 1

      Найдите точку на функции.

      $$ f\left(\frac \pi 6\right) = \cos \frac \pi 6 = \frac{\sqrt 3} 2 $$

      Точка $$\left(\frac\pi 6, \frac{\sqrt 3} 2\right)$$.

      Шаг 2

      Найдите значение производной при $$x = \frac \pi 6$$.

      $$ f'(x) = -\sin x\longrightarrow f’\left(\frac \pi 6\right) = -\sin\frac\pi 6 = -\frac 1 2 $$

      Наклон касательной равен $$m = -\frac 1 2$$. Поскольку мы ищем линию, перпендикулярную касательной, мы хотим использовать $$m = 2$$.

      Шаг 3

      Найдите уравнение прямой, проходящей через точку $$\left(\frac \pi 6, \frac{\sqrt 3} 2\right)$$ с наклоном $$m =2$$.

      $$ \начать{выравнивать*} y -y_1 & = m(x-x_1)\\[6pt] y — \frac{\sqrt 3} 2 & = 2\left(x — \frac\pi 6\right) \конец{выравнивание*} $$

      Отвечать

      Прямая, нормаль к функции в точке $$x = \frac \pi 6$$, равна $$y — \frac{\sqrt 3} 2 = 2\left(x — \frac \pi 6\right)$$.

      Для справки, вот график функции и найденная нами нормаль.

      Продолжайте практиковать задачи

      Ошибка: Нажмите «Не робот», затем повторите попытку загрузки.

      .

    Добавить комментарий

    Ваш адрес email не будет опубликован. Обязательные поля помечены *